Sie sind auf Seite 1von 62

Cvs 1

q 48

A 58-year-old Caucasian male presents to the emergency department complaining that he is coughing up blood and has trouble breathing, especially when he is engaged in activity. His medical history is significant for diverticulosis and a surgically repaired inguinal hernia. He is married with two children and does not smoke cigarettes, drink alcohol, or use recreational drugs. Vital signs include temperature of 37.0C (98.6F), blood pressure of 126/82 mm Hg, pulse of 76/min, and respirations of 14/min. His head, eye, ear, nose, and throat examination are unremarkable. Chest auscultation demonstrates crackles in the middle and upper lobes of both lungs. What will cardiac auscultation reveal if there is prolapse of his mitral valve? B. C. D. E. A. Holosystolic murmur Systolic crescendo-decrescendo ejection murmur Mid systolic click followed by late systolic murmur Loud S1 and low-pitched diastolic murmur loudest at apex Muffled heart sounds and low-pitched diastolic murmur loudest at apex

This subscription is licensed to user: roopika only User ID: 123489 Explanation: Mitral valve prolapse (MVP) is found in 2-5% of adults and is the most common valvular abnormality among the residents of industrialized nations. The condition is typically caused by redundant valve tissue at the valve ring (Barlow's syndrome) and may be associated with mild mitral regurgitation, though left ventricular function remains normal. MVP causes a mid-to-late systolic click that may be followed by a late systolic murmur most easily detected over the left ventricle (Choice C). An apical holosystolic murmur (Choice A) may be due to mitral regurgitation, which is most often secondary to myocardial infarction, mitral valve prolapse, rheumatic heart disease, or coronary artery disease. A systolic crescendo-decrescendo ejection murmur in the left upper sternal border (Choice B) characterizes pulmonic stenosis, which is most often secondary to congenital malformation, rheumatic heart disease, or carcinoid. Mitral stenosis produces a low-pitched, rumbling diastolic murmur best heard over the apex when the patient is lying in a left lateral decubitus position (Choice D). Muffled heart sounds (Choice E) are a classic indication of cardiac tamponade, and are not commonly associated with heart murmurs.

Educational Objective: Mitral valve prolapse causes a mid-to-late systolic click that may be followed by a late systolic murmur most easily detected over the left ventricle. 82% of people answered this question correctly; This subscription is licensed to user ID: 123489 only

A 63-year-old Caucasian woman comes to the physician for a follow-up visit. Due to a recent episode of pulmonary embolism, she has been on chronic anticoagulation therapy with warfarin 6 mg/day for the past four months. You have been following her PT/INR regularly. She shows you a report, which details that she was recently admitted to the hospital because of an episode of paroxysmal atrial fibrillation. Five days ago, she was discharged on amiodarone. Her other medications include ramipril and metoprolol. Her vital signs are within normal limits. Examination shows no abnormalities. Which of the following is the most effective strategy to decrease this patient's risk of adverse drug reactions? B. C. D. E. A. Decrease amiodarone dose Monitor PT and INR at least once a week for the next 3 months Increase warfarin dose to 7 mg/day Decrease warfarin dose to 3 mg/day Decrease warfarin dose to 4.5 mg/day

This subscription is licensed to user: roopika only User ID: 123489 Explanation: It is extremely important to identify which drugs interact with warfarin in an anticoagulated patient. Amiodarone is one of those drugs. This medication will prolong the action of warfarin in a predictable manner. For this reason, it is advisable to decrease the warfarin dose by approximately 25% (in this case, from 6 mg/day to 4.5 mg/day) to avoid excessive anticoagulation. (Choice A) The amiodarone dose does not need to be decreased. (Choices C and D) An increase of warfarin to 7 mg/day will lead to warfarin toxicity, and a decrease to 3 mg/day can produce sub-therapeutic anticoagulation. (Choice B) Monitoring of PT and INR is needed, but only after the warfarin dose is decreased appropriately. In a stable patient, monthly measurement of PT/INR is sufficient.

Educational Objective: Amiodarone interacts with warfarin, thereby prolonging its effect and increasing the risk of bleeding. Its clinical response is predictable; it is known that the warfarin dose needs to be decreased by approximately 25% in order to avoid overcoagulation. 16% of people answered this question correctly; This subscription is licensed to user ID: 123489 only

A 50-year-old Caucasian man is brought to the emergency department by the paramedics after an episode of chest pain of sudden onset. He was mowing the lawn on a cold winter day, when he suddenly felt severe chest pressure over the left side. The pain was constant, radiating to the left jaw, and was not relieved by rest or nitroglycerine spray in the ambulance. He has a history of controlled hypertension. He has no other significant past medical or family history. He smokes occasionally, especially over the weekends. He denies any alcohol intake. His temperature is 36.2C (97F), blood pressure is 86/57 mmHg, heart rate is 42/min, and respiratory rate is 16/min. His stat electrocardiogram reveals a sinus bradycardia with a 3 mm ST segment elevation in leads II, III and AVF. Which of the following is the most appropriate initial step in the management of this patient? B. C. D. E. A. IV normal saline 300 mL over the next 30 minutes Intravenous Dobutamine Intravenous atropine sulfate Temporary pacemaker insertion Synchronized cardioversion

This subscription is licensed to user: roopika only User ID: 123489 Explanation: The patient has developed sinus bradycardia as a complication of acute inferior wall MI. Sinus bradycardia is seen in approximately 15 to 25% of patients with an acute inferior wall MI. The heart rate is primarily controlled by the intrinsic pacemaker of the heart, and is located at the SA node. Blood is supplied to the SA node by the right coronary artery in 60 to 70% of the population; hence, its interruption in patients with an inferior wall MI can cause sinus bradycardia. Aside from this, sinus bradycardia can also be caused by an increased vagal tone during and after an acute MI, and by a reperfusion arrhythmia after thrombolysis. Sinus bradycardia after an acute inferior wall MI is usually transient, and typically resolves within the next 24 hours. Patients are usually asymptomatic and require no further treatment; however, severe bradycardia can result in a low cardiac output, thereby causing hypotension and subsequent hemodynamic compromise, as well as worsening of

ongoing ischemia in the setting of acute myocardial infarction with reduced ventricular function. If treatment becomes necessary, intravenous atropine sulfate (0.6 to 1.0 mg) can be given to reverse the bradycardia and improve the cardiac output and hypotension. (Choice A) Intravenous fluids are usually required in patients with right ventricular infarction, which causes reduced left ventricular filling pressures and hypotension. The primary cause of hypotension in this patient is severe bradycardia resulting from an acute inferior wall MI. (Choice B) Dobutamine is a beta-1 agonist that can be used as an inotropic agent in patients with cardiogenic shock; however, since it can produce vasodilation and further hypotension, it should not be given to hypotensive patients. (Choice D) Sinus bradycardia after an acute MI is usually transient and resolves within the next 24 hours. Temporary cardiac pacing is required only if the bradycardia is persistent and symptomatic in spite of IV atropine use. (Choice E) Synchronized cardioversion has no role in the management of patients with severe bradycardia. Educational Objective: Although symptomatic severe bradycardia after an acute inferior wall MI is usually transient, it should be treated with intravenous atropine sulfate. 37% of people answered this question correctly; This subscription is licensed to user ID: 123489 only

A 47-year-old Caucasian man comes to the office for a follow-up visit. He has no complaints. His medical problems include obesity, hypercholesterolemia and hypertension. He has smoked one pack of cigarettes daily for 30 years. His father had a myocardial infarction when he was 50 years old. His medications include lovastatin, enalapril and hydrochlorothiazide. His vital signs are within normal limits. Examination reveals no abnormalities. He is concerned about the possibility of having myocardial infarction as his father did. The cardiologist ordered the measurement of plasma fibrinogen levels, which was 3.5 g/L (normal values < 2.39 g/L for smokers). The lipid profile and serum chemistry were also obtained. The results show the following: Serum chemistrySerum Na 140 mEq/L Serum K 4.0 mEq/L Chloride 103 mEq/L Bicarbonate 24 mEq/L BUN 20 mg/dL Serum creatinine 0.8 mg/dL

Calcium 10.0 mg/dL Blood glucose 98 mg/dL Lipid profileTotal cholesterol 268 mg/dL HDL Cholesterol 56 mg/dL LDL Cholesterol 190 mg/dL Triglycerides 148 mg/dL Which of the following interventions can avoid a further rise of the patient's fibrinogen levels? B. C. D. E. A. Refer the patient to a smoking cessation program Stop lovastatin and start pravastatin Stop hydrochlorothiazide Stop enalapril and start lisinopril Start the patient in an exercise program

This subscription is licensed to user: roopika only User ID: 123489 Explanation: Fibrinogen has been associated with an increased cardiovascular risk. Individuals with levels greater than 3.43 g/L duplicate their risk of having a myocardial infarction. Fibrinogen levels over 2.77 g/L are considered high. Nevertheless, routine determination of fibrinogen levels is not standard practice. Although drug therapy to decrease fibrinogen has not proven to prevent cardiac events, current knowledge suggests that measures should be taken to avoid further increases. Statins affect fibrinogen metabolism: lovastatin is associated with high levels; atorvastatin can increase fibrinogen as much as 46%, while pravastatin and simvastatin cause no change. According to current data, lovastatin should not be used if fibrinogen levels are higher than 2.77 g/L because this medication will worsen hyperfibrinogenemia. Changing lovastatin for pravastatin will help to decrease fibrinogen levels. (Choice A) Smokers have fibrinogen concentrations, which are 0.3 g/L higher than nonsmokers. Smoking cessation decreases fibrinogen level, but it takes approximately 20 years to be effective. (Choices C and D) Neither hydrochlorothiazide nor enalapril alter fibrinogen levels. These medications need to be continued in order to maintain an adequate blood pressure control. There is no advantage in taking lisinopril instead of enalapril in this case. (Choice E) Although exercise is beneficial to prevent cardiovascular events and to increase HDL levels, it has no effect on fibrinogen levels.

Educational Objective: In prospective studies of healthy patients, those with higher plasma fibrinogen concentrations developed coronary heart disease more frequently. Fibrinogen levels decrease only 20 years after smoking cessation; while certain drugs such as atorvastatin and lovastatin can produce hyperfibrinogenemia. Even though it is uncertain how dangerous hyperfibrinogenemia can be, it is currently recommended to maintain fibrinogen levels as low as possible and to discontinue any medication that can increase them, unless the benefits outweigh the risks. 16% of people answered this question correctly; This subscription is licensed to user ID: 123489 only

A 43-year-old white male comes to your clinic for a routine follow-up visit. He has been a patient of yours for the past five years. He does not have any significant past medical history except for GERD. He is not currently taking any prescription medications. His father and brother (in his early 40's) have both had a myocardial infarction. He denies any cigarette smoking or alcohol abuse in the past. His blood pressure during his last visit two months ago was 142/86 mmHg. His fasting lipid panel at that time showed the following values: total cholesterol 204 mg/dL triglycerides 106 mg/dL HDL 44 mg/dL LDL 126 mg/dL He says he has faithfully followed the advice you gave two months ago, when you explained the need for lifestyle modifications such as moderate exercise and low salt diet. His blood pressure during this visit is 146/84 mmHg. He denies any symptoms whatsoever, and his examination is unremarkable. Which of the following statements is true regarding this patient's further management? A. Given his strong family history of coronary artery disease, an exercise stress test in the near future is highly recommended B. This patient should be started on a daily Baby Aspirin C. The patient should be started on a statin to lower his LDL levels to a goal of 100 mg/dL or less D. Thiazide diuretics are the recommended anti-hypertensive agents at this point E. The patient should be started on a low cholesterol diet and his lipid levels rechecked in 6 months F. Patient should be started on a low-dose ACE inhibitor, and the dose titrated up depending on the BP response This subscription is licensed to user: roopika only User ID: 123489

Explanation: The patient in this vignette clearly meets the diagnosis of Stage I hypertension. According to the new JNC VII guidelines, thiazide diuretics are the initial drug of choice for the treatment of essential hypertension. (Choices A and B) Although the patient has a significant and concerning family history, there is no evidence from the history and examination which suggests that the patient has coronary artery disease. In the absence of any symptoms, an exercise stress test is not recommended. For the same reason, in the absence of any known coronary disease, aspirin therapy has not been shown to be useful and may even be harmful to the patient. (Choices C and E) NCEP guidelines recommend that LDL levels should be kept below 100mg/dL in patients with known coronary disease, prior history of myocardial infarction, peripheral and cerebral vascular disease, and inpatients with diabetes mellitus (CAD equivalent). In the absence of the abovementioned, the patient's current lipid profile is considered normal; therefore, he does not need treatment for hypercholesterolemia or more frequent monitoring. (Choice F) ACE inhibitors, calcium channel blockers, or beta-blockers are added only as second line agents for hypertension if it is not controlled by single agents. Educational Objective: According to the new JNC VII guidelines, thiazide diuretics are the initial drugs of choice for the treatment of essential hypertension. 36% of people answered this question correctly; This subscription is licensed to user ID: 123489 only

A 52-year-old man comes to the emergency department (ED) and complains of severe substernal chest pressure for the past hour. His symptoms started when he was helping his sister move to her new home. He has had such similar episodes with exertion in the past, but those last only for a few minutes and are relieved with rest and sublingual nitroglycerin. The chest pain now radiates to his neck and left side of the jaw, and is associated with mild dizziness and shortness of breath. He took 3 nitroglycerin tablets sublingually from a six month-old bottle, but this did not relieve his pain at all. His current daily medications include metoprolol, lisinopril, atorvastatin, and aspirin. His vital signs are within normal limits, except for tachycardia. His lungs are clear on auscultation bilaterally. Cardiac examination reveals a prominent point of maximal impulse that is displaced downwards, and to the left side. Auscultation reveals regular and normal heart sounds with an audible S4. There are no audible murmurs. The rest of the physical examination is within normal limits. An electrocardiogram in the ED reveals

non-specific ST-T wave changes with prominent QRS complex in the lateral leads. What is the most appropriate next step in the management of this patient? A. Urgent cardiac catheterization B. Admit the patient to a telemetric unit and do serial cardiac enzymes C. Order a stress echocardiogram D. Order a radionuclide perfusion scan E. Provide him with a new prescription of nitroglycerin and ask him to obtain an outpatient stress test This subscription is licensed to user: roopika only User ID: 123489 Explanation: The patient in the above vignette has presented to the emergency room with symptoms suggestive of acute coronary syndrome. He has a prior history of stable angina pectoris. However, the worsening symptoms of angina lasting for a longer duration and unrelieved with rest or nitrogycerin is suspicious for acute coronary syndrome. He most likely has either unstable angina or non-ST elevation myocardial infarction. The two conditions are mainly distinguished on the basis of the presence or absence of an elevation in the serum biomarkers of myocardial damage (troponin I & T, creatine kinase and its MB-isoform). Patients with NSTEMI have ischemic symptoms suggestive of acute coronary syndrome with elevated cardiac enxyme levels. On the other hand, patients with unstable angina have similar symptoms, with or without EKG changes, and with no rise in the cardiac enzyme levels. The symptoms of this patient are suggestive of acute coronary syndrome. Although he has a prior history of stable angina pectoris, his worsening symptoms (e.g., angina lasting for a longer duration and unrelieved with rest or nitrogycerin) require admission into a telemetric unit and further work-up with serial measurements of cardiac enzyme levels in order to adequately diagnose his current condition. (Choice A) Urgent cardiac catheterization is the preferred therapy in patients with ST segment elevation myocardial infarction (STEMI). It is not indicated in patients with NSTEMI. (Choices C and D) Stress testing (either with stress echocardiogram or radionuclide perfusion scan) is generally performed as a part of risk stratification prior to the discharge to aid in treatment decisions in patients with unstable angina. High-risk patients identified on stress testing can be referred for a coronary angiography or percutaneous coronary intervention. On the other hand, patients with NSTEMI should be referred for early percutaneous coronary intervention without prior stress testing. (Choice E) Discharging the patient with acute coronary syndrome is inappropriate and can have potentially disastrous outcomes.

Educational Objective: All patients presenting with acute coronary syndrome (unstable angina and non-ST elevation MI) should be admitted to the hospital for close observation, serial measurement of cardiac enzymes and antithrombotic therapy. 63% of people answered this question correctly; This subscription is licensed to user ID: 123489 only

A 38-year-old Caucasian female presents to your office for a routine check-up. She has no present complaints. Her past medical history is significant for hypertension and diabetes mellitus, type II, both of which are controlled with low-dose hydrochlorothiazide and glyburide, respectively. She smokes one pack of cigarettes daily, and consumes alcohol occasionally. Her blood pressure is 130/85 mmHg and heart rate is 85/min. Physical examination reveals moderate obesity (BMI = 30 kg/m2). Her fasting blood glucose level is 120 mg/dL and HbA1C level is 6.7%. Which of the following factors is the single most important predictor of future cardiovascular events for this woman? A. B. C. D. E. F. G. Obesity Age Hypertension Smoking Diabetes mellitus Gender Alcohol consumption

This subscription is licensed to user: roopika only User ID: 123489 Explanation: Diabetes mellitus is the single most important predictor of adverse cardiovascular outcomes. Due to the very high prevalence of atherosclerotic vascular disease and increased risk of future cardiovascular events in diabetics, diabetes mellitus is considered as an equivalent of coronary heart disease (CHD). Diabetes has even more prognostic significance for women. Moreover, any additional risk factor in patients with diabetes mellitus (e.g., hypertension, smoking) has a synergistic effect, greatly increasing the risk. In this patient, glycemic status seems to be well controlled, but diabetes mellitus remains as the most important risk factor. The role of glycemic control in the development of macrovascular disease in patients with type-2 diabetes is not firmly established. (Choices F and B) Female gender traditionally is considered protective until menopause. (Choice C) Hypertension is a well-established risk factor. The blood pressure should be kept below 130/85 mmHg in diabetics, unlike non-diabetic individuals whose target blood pressure is below 140/90 mmHg.

(Choice D) Smoking is an important modifiable risk factor. The risk of cardiovascular events declines rapidly after smoking cessation, and approaches that of non-smokers in several years. (Choice A) Obesity is an independent risk factor for cardiovascular disease, but it is also associated with an unfavorable metabolic profile. (Choice G) Moderate alcohol consumption is associated with protection against adverse cardiovascular outcomes. Educational Objective: Diabetes mellitus is the single most important predictor of adverse cardiovascular outcomes, especially in women. It is considered to be an equivalent of coronary heart disease (CHD). The role of glycemic control in the development of macrovascular disease in patients with type-2 diabetes is not firmly established. 33% of people answered this question correctly; This subscription is licensed to user ID: 123489 only

A 62-year-old Caucasian man comes to the emergency department (ED) and complains of crushing substernal chest pain. The pain started one hour ago while he was shoveling snow on his driveway. At the onset of his pain, he took nitroglycerin sublingually, but the pain did not get any better and he was taken to the ED by his son. He has a history of coronary artery disease, hypertension, hyperlipidemia, and osteoarthritis. He is on daily aspirin, metoprolol, lisinopril, atorvastatin, and naproxen as needed. In the ED, his electrocardiogram reveals a normal sinus rhythm with no acute ST-T wave changes. He is given morphine for his pain, started on nitroglycerin drip, and admitted to the telemetry floor. The next day, an adenosine thallium stress test reveals a moderate size reversible perfusion defect in the lateral wall of the left ventricle. Which of the following coronary vessels is most likely to have a significant occlusion? B. C. D. A. Left circumflex coronary artery Left anterior descending coronary artery Left main coronary artery Right coronary artery

This subscription is licensed to user: roopika only User ID: 123489 Explanation: In all patients with known or suspected coronary artery disease, exercise or pharmacological stress testing with radionuclide myocardial perfusion imaging (thallium or technetium-99m) is useful for confirming the diagnoses and for risk stratification. The

pharmacologic stress agents (adenosine and dipyridamole) act by producing coronary vasodilatation and increasing the coronary flow rate and velocity. In normal coronary vessels, the resulting vasodilation increases the blood flow; however, in areas with severe stenosis, there is already a compensatory microvascular dilatation at rest to maintain normal blood flow, so no further increase in the flow occurs. The resulting heterogenous blood flow due to the stenotic or occluded region is detected by radionuclide imaging studies as a perfusion defect. After establishing the presence of a defect, the next step is to localize the affected myocardial region in order to identify the culprit vessel. All coronary arteries supply a specific region of the ventricle. In this case, the patient's adenosine thallium stress test revealed a perfusion defect in the lateral wall of the left ventricle. The culprit vessel is most probably the left circumflex artery, which runs laterally in the left atrioventricular groove to supply the lateral and posterolateral parts of the left ventricle. (Choices B and C) The left main coronary artery arises from the left coronary ostium and branches off into the left anterior descending (LAD) artery and left circumflex artery. The LAD artery runs along the anterior interventricular groove and supplies the anterior wall of the left ventricle. Diagonal branches from the LAD artery also run downward to supply the anterolateral wall of the left ventricle. (Choice D) The right coronary artery runs in the right atrioventricular groove to supply the right ventricle and inferoposterior walls of the left ventricle. Educational Objective: The left circumflex coronary artery runs in the left atrioventricular groove to supply the lateral wall of the left ventricle. 64% of people answered this question correctly; This subscription is licensed to user ID: 123489 only

A 55-year-old Hispanic woman comes to the emergency department with symptoms of intermittent substernal chest discomfort for the past 18 hours. Her other medical problems include hyperlipidemia, hypertension, diabetes mellitus- type 2, obesity, and hypothyroidism. She denies any history of smoking or alcohol abuse. Her medications include metformin, lisinopril, metoprolol, amlodipine, simvastatin, and levothyroxine. She is allergic to penicillin and sulfonamides. Her temperature is 37.2 C (99 F), blood pressure is 140/90 mmHg, pulse is 92/min, and respirations are 18/min. Her pulse oximetry reading is 99% on 2-liters oxygen. Examination shows normal jugular venous pressure, clear lung fields bilaterally, normal first and second heart sounds, and a grade 2/6 ejection systolic murmur in the right second intercostal area. All of her lab studies are normal, except an elevated blood glucose of 215 mg/dL, CK-MB and troponin-T. The electrocardiogram demonstrates deeply inverted T waves in the lateral leads. She is

admitted to the hospital and started on standard therapy. Which of the following medications should be discontinued during her current hospitalization? B. C. D. E. A. Lisinopril Metoprolol Amlodipine Simvastatin Metformin

This subscription is licensed to user: roopika only User ID: 123489 Explanation: According to the ACC/AHA guidelines, no calcium channel blocker has been shown to reduce the mortality rate of patients with acute myocardial infarction, and this drug may even be harmful in certain patients. The only indications of calcium channel blockers in the setting of acute myocardial infarction include intolerance to beta blockers, postinfarction angina refractory to beta blockers and nitrates, and rapid atrial fibrillation with contraindications to beta-blocker use. (Choice A) Metformin is the drug of choice for treating type 2 diabetes in this obese female; it should only be withheld temporarily and not discontinued if any intravenous dye-based study or intervention is planned. (Choices B and C) Beta-blockers and ACE inhibitors (and also angiotensin receptor blockers, if ACE inhibitors are contraindicated) are a part of the standard therapy for the treatment of patients with acute myocardial infarction. A large number of clinical studies have clearly demonstrated a survival benefit of treatment with both of these agents. (Choice D) There is no reason to stop simvastatin in this patient, unless the patient develops adverse side effects with its use. Educational Objective: Calcium channel blockers can be harmful in patients with acute MI. Beta blockers, ACE inhibitors, and statins have significant effects on the secondary prevention of coronary artery disease, and should be continued indefinitely unless absolutely contraindicated. 24% of people answered this question correctly; This subscription is licensed to user ID: 123489 only

A 68-year-old Caucasian man comes to the physician because of recurrent episodes of dizziness. His medical problems include hypertension, coronary artery disease (CAD),

asthma, diabetes mellitus type-2, and hypercholesterolemia. He is retired and lives alone. He does not use tobacco, alcohol, or drugs. His family history is not significant. His medications include aspirin, enalapril, isosorbide mononitrate, simvastatin, glyburide, and metformin. His temperature is 36.7 C (98 F), blood pressure is 125/80 in decubitus and 100/70 mmHg in the standing position, pulse is 86/min. in decubitus and 92/min. while standing, and respirations are 14/min. Examination shows no abnormalities. Which of the following is the most effective strategy to manage this patient's symptoms? B. C. D. E. A. Stop enalapril. Stop isosorbide. Stop simvastatin. Order a CT scan of the head . Start meclizine.

This subscription is licensed to user: roopika only User ID: 123489 Explanation: Polypharmacy is defined as the use of five or more drugs in a single patient for different medical conditions. It is very common in the elderly. Knowledge of the various drug interactions, as well as their adverse effects, is very essential to the primary care provider. For instance, although the usual treatment of coronary artery disease (CAD) involves beta-blockers, this was not given to the patient due to his other medical problems (diabetes and asthma). This patient has multiple medical conditions, for which he has the following medications: aspirin, enalapril, isosorbide mononitrate, simvastatin, glyburide, and metformin. He currently complains of dizziness and has orthostatic hypotension, which is associated with the use of diuretics, alpha-blockers, or nitrates. Isosorbide dinitrate use has been associated to dizziness (in 4% of patients) and to orthostatic hypotension (in 1% of patients). Isosorbide use must therefore be stopped in this patient. (Choice A) Enalapril is an antihypertensive drug which has not been associated with orthostatic hypotension as frequently as the abovementioned drugs. (Choice C) Simvastatin use can cause dizziness as frequently as a placebo; furthermore, its use has not been associated with orthostatic hypotension. (Choice D) The diagnosis has been clearly established. There is no need for further studies, such as a CT scan or MRI of the head, and a neurology referral. (Choice E) Meclizine can have some symptomatic benefit in patients with vestibular disease. It will not be helpful in cases of dizziness associated with orthostatic hypotension. Educational Objective:

Polypharmacy is frequent in the elderly. Orthostatic hypotension is not uncommon in this setting and is usually related to the use of diuretics, alpha-blockers, or nitrates. 83% of people answered this question correctly; This subscription is licensed to user ID: 123489 only

A 47-year-old Chinese-American man comes to the office for a routine follow-up visit. His medical problems include hypertension, diabetes mellitus type-2, and hypercholesterolemia. He has smoked one pack of cigarettes daily for 20 years. His medications include aspirin, ramipril, metformin, and pravastatin. Examination shows no abnormalities. His temperature is 36.7 C (98 F), blood pressure is 140/82 mmHg, pulse is 74/min., and respirations are 14/min. His LDL cholesterol level is 140 mg/dL. Which of the following is the most effective strategy to decrease this patient's risk for developing acute coronary artery disease? B. C. D. E. A. Stopping smoking Strict diabetes control Decreasing LDL cholesterol Control of hypertension Exercise

This subscription is licensed to user: roopika only User ID: 123489 Explanation: Randomized, prospective, double-blinded, and placebo-controlled studies have proven that the risk of coronary artery disease (CAD) can be significantly reduced by decreasing LDL cholesterol and, to a lesser degree, by adequately controlling the blood pressure. (Choice E) Although exercise is advisable, it is less effective for acute CAD prevention than the other measures. Recent studies even show that it may not decrease endothelial coronary damage or decrease atheroma size. (Choice A) Stopping smoking is important because it will decrease endothelial damage, however the benefit is clearly lower than control of hypertension and LDL levels. (Choice B) Diabetes, especially if uncontrolled, is an important risk factor for endothelial damage. It is currently considered as an equivalent of CAD because of the damage that it produces in the coronary artery endothelium; however, diabetes control by itself decreases the risk of cardiovascular events by a lower proportion only. LDL levels in these patients should also be decreased (to less than 100 mg/dL) in order to effectively prevent cardiovascular events.

(Choice D) Controversies currently exist on which intervention is more effective: decreasing LDL cholesterol or controlling hypertension. At present, evidence-based studies support the notion that decreasing LDL cholesterol is the more effective intervention than hypertension control. Most studies have also shown that treatment of mild hypertension has not been effective for CAD prevention. Educational Objective: Multiple risk factors increase the risk of acute CAD. Randomized, prospective, doubleblinded, and placebo-controlled studies have proven that the risk can be significantly reduced by decreasing LDL cholesterol and, to a lesser degree, by adequately controlling the blood pressure (in diabetic patients, as well as in patients with previous CAD, a blood pressure lower than 130/80 mmHg is recommended). The contribution of other interventions such as smoking cessation, diabetes control, exercise, and HDL increase is also beneficial, but not as critical as the first two measures. 24% of people answered this question correctly; This subscription is licensed to user ID: 123489 only

The following vignette applies to the next 2 items A 38-year-old healthy Caucasian man is brought to the emergency department because of sudden onset of shortness of breath and diaphoresis. He denies fever, chills, cough or abdominal pain. He has no other medical problems. He had surgery for bilateral inguinal hernia when he was 16. He does not use tobacco, alcohol or illicit drugs. He takes no medication and has no known drug allergies. His blood pressure is 110/60 mm Hg, pulse is 116/min and respirations are 28/min. He is in marked respiratory distress. Pallor and diaphoresis are noted. His skin is velvety and has multiple scars. On auscultation of the heart, an early, decrescendo, systolic murmur at the cardiac apex is heard; the murmur decreases with Valsalva maneuver, and increases with the grip maneuver, radiating to the axilla. The first sound is barely audible; the second heart sound is normal. A fourth heart sound is also present. There are bilateral crackles in both lungs. Jugular venous distention and hepatojugular reflux are present. The abdomen is soft, non-tender and non-distended. The neurologic examination reveals no abnormalities. The initial EKG shows sinus tachycardia with occasional premature ventricular complexes. The chest x-ray reveals no cardiomegaly, but bilateral alveolar infiltrates and hilar prominence are present. Item 1 of 2 Which of the following is the most likely cause of his condition? B. A. Acute myocardial infarction Rupture of chordae tendineae

C. D. E.

Pulmonary embolism Infective endocarditis Papillary muscle rupture

This subscription is licensed to user: roopika only User ID: 123489 Explanation: The patient presents with signs and symptoms of acute heart failure. His EKG findings reveal occasional premature ventricular complexes (PVC); there are no signs of ischemia or ventricular hypertrophy. His pathologic murmur (systolic murmur that is heard in the apex, radiates to the axilla, increases with the grip maneuver, and decreases with Valsalva) is characteristic of mitral regurgitation, which may lead to acute heart failure. Acute mitral regurgitation is usually characterized by a soft, decrescendo systolic murmur (can be early, midsystolic or holosystolic), a decreased first heart sound, and the presence of a fourth heart sound. Due to the acute nature of this condition, there is no evidence of left ventricular hypertrophy in the electrocardiogram or the chest x-ray. The four common causes of acute heart failure are papillary muscle rupture, infective endocarditis, rupture of chordae tendineae, and chest wall trauma with compromise of the valvular apparatus. The most common cause of isolated, severe acute mitral regurgitation in adults is rupture of chordae tendineae with or without associated myxomatous disease. The diagnosis is confirmed by echocardiography. (Choice A) Myocardial infarction can be complicated by acute mitral regurgitation when there is rupture of the papillary muscle; however, this patient does not have evidence of ischemia in the EKG. (Choice C) Patients with pulmonary embolism can develop acute cor pulmonale; however, in such a setting, the EKG will show right axis deviation, right bundle branch block or both. The above patient clearly has acute pulmonary edema. (Choice D) The patient does not have fever or risk factors for bacterial endocarditis. (Choice E) Spontaneous papillary muscle rupture usually presents in elderly people who have acute chest pain or as a complication of myocardial infarction. None of these are present here. Educational Objective: Rupture of chordae tendineae should be suspected in healthy individuals who develop flash pulmonary edema (heart failure) associated with an acute mitral regurgitation. The differential diagnosis of this condition includes infective endocarditis, papillary muscle rupture secondary to ischemia, and mitral valve rupture secondary to trauma. 50% of people answered this question correctly; This subscription is licensed to user ID: 123489 only

Item 2 of 2 The patient is feeling better after receiving the appropriate therapy. A more detailed physical examination reveals the presence of scoliosis and pes planus. The cardiac enzymes were negative. Blood gas analysis reveals respiratory alkalosis. Complete blood cell count and serum chemistry results are unremarkable. The patient is scheduled for echocardiography. Which of the following is the most probable diagnosis? B. C. D. E. A. Coronary artery disease Rheumatic fever Marfan syndrome Ehlers-Danlos syndrome Thyrotoxicosis

This subscription is licensed to user: roopika only User ID: 123489 Explanation: The patient has features compatible with Ehlers-Danlos syndrome. This condition can cause a myxomatous degeneration of the mitral valve, leading to acute rupture of chordae tendineae. Pes planus and scoliosis are frequent, though not specific findings. The skin can be velvety or thin, and is usually covered with multiple characteristic "cigarettepaper" scars, due to its frailty and easy bruisability. A past history of hernias and mitral valve prolapse is not uncommon. Joint hypermobility and skin hyperextensibility ("rubber man syndrome") can be dramatic in severe cases. (Choice A) There is no evidence of ischemic heart disease. Troponins and CK-MB are normal. The EKG shows no ischemic changes. (Choice E) Thyrotoxicosis can lead to acute, high-output, heart failure; however, this will not explain the mitral regurgitation. Skin changes of thyrotoxicosis can be confounded with those found in Ehlers-Danlos, but other characteristic signs and symptoms are lacking. (Choice B) Rheumatic fever is a common cause of mitral regurgitation in young individuals; however, the course of the disease is chronic, and the patients usually have evidence of left ventricular and atrial hypertrophy in the EKG. (Choice C) Marfan syndrome can affect the valvular apparatus, causing degeneration of the mitral and aortic valves. The presentation is usually chronic, progressive mitral regurgitation, and rarely, acute regurgitation due to ruptured chordae tendineae. This patient does not have arachnodactyly, loose joints or increased arm span, which are characteristic of Marfan syndrome.

Educational Objective: Marfan or Ehlers-Danlos syndrome must be suspected in patients with connective tissue abnormalities and an acute mitral regurgitation secondary to chordae tendineae rupture, although a primary, pre-existing mitral valve prolapse (MVP) is the most common cause. Some cases may be idiopathic (individuals who experience rupture of the chordae tendineae without previous MVP or connective tissue disease), but an etiology can be found in most occasions. 52% of people answered this question correctly; This subscription is licensed to user ID: 123489 only

A 54-year-old heavyset Caucasian man presents to the emergency department complaining of chest pain that began approximately thirty minutes ago while he was mowing the lawn. He is panting and sweating heavily. He describes the chest pain as intense pressure, "like a pickup truck is right on top of me." The pain radiates to his neck, jaw, and left arm. He has never experienced an episode like this before. He has been diagnosed with diabetes mellitus, hypertension, hypercholesterolemia, peptic ulcer disease, and onychomycosis. He smokes one pack of cigarettes per day and consumes 1-3 alcoholic drinks per night. He denies usage of recreational drugs. Vital signs include temperature of 37.7C (99.9F), blood pressure of 142/90 mm Hg, pulse of 114/min, and respirations of 21/min. There is diffuse mild chest tenderness to palpation. No edema of the extremities is evident and the remainder of the examination is unremarkable. He was given oxygen and aspirin while en route to the hospital. Nitroglycerin is now administered, which provides moderate pain relief. Which of the following findings on electrocardiogram most strongly indicates the need for thrombolytic therapy? B. C. D. E. A. ST depression >1mm in the lateral leads ST elevation >1mm in the lateral leads T wave inversion in the inferior leads Poor R wave progression QT prolongation

This subscription is licensed to user: roopika only User ID: 123489 Explanation: Patients stricken with chest pain suggestive of an acute myocardial infarction are eligible for thrombolytic therapy if they present within 12 (and sometimes 24) hours of symptom onset and if an electrocardiogram demonstrates ST elevations >1mm in two contiguous leads (Choice B). It is important to administer nitroglycerin in these patients before performing the electrocardiogram to rule out coronary vasospasm. Additional candidates for thrombolytic therapy include individuals with persistent symptoms of new left bundle

branch block. There is no indication that thrombolytic therapy is of benefit in patients with non-ST elevation acute coronary syndrome. Contraindications for thrombolytic therapy include evidence of active bleeding, adverse intracerebral history (history of hemorrhage, ischemic stroke, or neoplasm), systolic blood pressure greater than 180 mm Hg, recent trauma, or drug allergy. ST depression >1mm (Choice A) is commonly caused by ischemia, "strain," digitalis effect, and hypokalemia or hypomagnesemia. ST segment depression is not an indication for thrombolysis unless it is evidence of a posterior or dorsal MI. T wave inversion (Choice C) commonly occurs in myocardial infarction, angina, and ventricular hypertrophy. However, T wave inversion rarely indicates ischemia if the QRS is also negative in those leads. Poor R wave progression (Choice D) occurs when the R wave in leads V1 through V4 remains the same size or increases very little. Common pathologic causes of poor R wave progression include LVH, RVH, COPD, anterior infarction, conduction defects, and cardiomyopathy. The QT interval is considered prolonged (Choice E) if it measures more than half the R-R interval, though measurement of the interval is of little clinical significance if the patient is tachycardic. Common causes of QT prolongation include drugs (eg, antiarrhythmic agents and tricyclic antidepressants), electrolyte imbalances (eg, hypokalemia), and CNS catastrophes (eg, stroke, seizure). Educational Objective: Patients with chest pain suggestive of an acute myocardial infarction are eligible for thrombolytic therapy if they present within 12 hours of symptom onset and if an electrocardiogram demonstrates ST elevations >1mm in two contiguous leads. 83% of people answered this question correctly; This subscription is licensed to user ID: 123489 only

A 68-year-old white male with a history of paroxysmal atrial fibrillation, hypertension, and hypercholesterolemia is brought to the physician's office by his daughter because of nausea, anorexia, and confusion. His daughter states that he was admitted at an outside hospital two weeks ago because of uncontrolled hypertension and palpitations. He was started on a new medication. Prior to the recent hospitalization, he was taking digoxin, warfarin, simvastatin, and hydrochlorothiazide. His physical examination shows an irregular heart rate, but is otherwise unremarkable. His blood pressure is 130/80 mmHg, pulse rate is 50/minute and irregular, temperature 37.2C (99F), and respirations are 14/minute. Which of the following is the most likely medication that is contributing to his symptoms?

B. C. D. E.

A. Enalapril Clonidine Valsartan Verapamil

Atenolol

This subscription is licensed to user: roopika only User ID: 123489 Explanation: This patient has developed classic early features of digoxin toxicity. Digoxin toxicity is characterized by nausea, vomiting, anorexia, fatigue, confusion, visual disturbances, and cardiac abnormalities. The most likely explanation of his digoxin toxicity is the recent addition of verapamil. Verapamil inhibits the renal tubular secretion of digoxin, resulting in almost 70-100% increase in serum digoxin levels. The other medications, which can cause digoxin toxicity, are quinidine, amiodarone, and spironolactone. (Choice B) Even though captopril can cause a slight elevation of digoxin levels, the other ACE inhibitors and ARBs usually do not cause digoxin toxicity. (Choice A) Beta-blockers may cause bradycardia when used with digoxin because of the added negative chronotropic action; however, this usually does not result in digoxin toxicity. (Choices C and D) The other medications in the other choices do not cause significant digoxin toxicity. Educational Objective: Verapamil, quinidine, and amiodarone can potentially cause digoxin toxicity. 28% of people answered this question correctly; This subscription is licensed to user ID: 123489 only

A 62-year-old Caucasian man comes to the emergency department because of shortness of breath, chest pain, and palpitations. His other medical problems include hypertension, obesity, chronic obstructive pulmonary disease, myocardial infarction, coronary artery bypass graft, and carotid artery stenosis. Electrocardiogram shows atrial fibrillation with a rapid ventricular response. The patient is connected to a cardiac monitor. While obtaining an IV access, the patient becomes unresponsive. The cardiac monitor still shows atrial fibrillation. Which of the following is the most appropriate next step in the management of this patient?

B. C. D. E.

A. Synchronized cardioversion Asynchronous cardioversion Start chest compressions Obtain arterial blood gas immediately Administer loading dose of diltiazem

This subscription is licensed to user: roopika only User ID: 123489 Explanation: Treatment of atrial fibrillation in a hemodynamically stable patient is different from one who is unstable. If the patient is hemodynamically unstable (low BP, patient not responding to commands), the treatment of choice is electrical cardioversion. If the patient is hemodynamically stable, the atrial fibrillation has to be categorized first as either an acute or a chronic process. If it is an acute process, the patient may undergo either cardioversion or rate control to convert his arrhythmia to sinus rhythm. If it is a chronic process, then the patient should be managed with rate control and anticoagulation. Cardioversion can be either electrical or chemical. Electrical cardioversion is indicated in hemodynamically unstable patients. Synchronized cardioversion (electrical shock administered in synchrony with the 'R' wave) is the procedure of choice. Electrical cardioversion should not be performed without 3-4 weeks of anticoagulation first in chronic atrial fibrillation (> 48 hours), as the risk of embolization is high. The required joules are about 100-200 joules. Atrial fibrillation is generally more resistant to convert to normal sinus rhythm than atrial flutter, and the energy required is much higher. If you choose to do chemical cardioversion, the drugs of choice are class III agents (amiodarone, sotalol, ibutilide, etc.). (Choice B) Asynchronized cardioversion is indicated for ventricular fibrillation. (Choice C) Chest compressions are indicated in patients with asystole. (Choice D) ABG is not indicated at this time. The patient must first be stabilized. (Choice E) Administering any form of rate control drug is not indicated in unstable patients. Educational Objective: If a patient with atrial fibrillation is hemodynamically unstable, the treatment of choice is synchronized electrical cardioversion. If the patient is hemodynamically stable, the atrial fibrillation has to be categorized first as either an acute or a chronic process. If it is an acute process, then you can choose to convert the patient to sinus rhythm by either cardioversion or rate control. If it is a chronic process, then it is best managed with rate control and anticoagulation. Rate control is best achieved with diltiazem or metoprolol.

66% of people answered this question correctly; This subscription is licensed to user ID: 123489 only

The following vignette applies to the next 2 items A 72-year-old Caucasian man comes to the physician's office because of palpitations. He has had these symptoms for the past few hours. He also complains of dyspnea on exertion and malaise for the past few months. His other medical problems include hypertension and degenerative joint disease. He has smoked one pack of cigarettes daily for 55 years. His medications include hydrochlorothiazide and fosinopril. His blood pressure is 140/90 mm Hg, pulse is 148/min, and respirations are 20/min. Physical examination reveals scattered rhonchi and wheezes in both lungs. The heart sounds and the rest of the examination are normal. The electrocardiogram (EKG) demonstrates tachycardia with narrow QRS complexes, different P wave morphologies, and variable PR segments and R-R intervals. A diagnosis of multifocal atrial tachycardia is made. Item 1 of 2 Which of the following is the most appropriate next step in management? B. C. D. E. A. Administer an intravenous bolus of digoxin Obtain a chest x-ray Administer intravenous amiodarone Check the arterial oxygen saturation Administer an intravenous dose of atenolol

This subscription is licensed to user: roopika only User ID: 123489 Explanation: Multifocal or multiform atrial tachycardia (MAT) is characterized by the presence of 3 or more P waves of different morphologies. The QRS complexes are narrow, while the PR segments and the R-R intervals are variable. The heart rate can reach up to 200 beats per minute. MAT is usually secondary to the following conditions: 1) hypoxia 2) chronic obstructive pulmonary disease (COPD) 3) hypokalemia 4) hypomagnesemia 5) coronary/ hypertensive/ valvular disease

6) medications (i.e., theophylline, aminophylline, isoproterenol) Hypoxia and COPD are the most common etiologies that affect mainly elderly patients. In this patient, hypoxia must first be ruled out since its rapid correction may eliminate the arrhythmia. (Choices A and C) Both digoxin and amiodarone are not effective therapeutic drugs for MAT. (Choice B) A chest x-ray will be useful in ruling out pneumonia or another lung pathology which may explain the patient's dyspnea, malaise and tachypnea; however, identifying hypoxemia via pulse oximetry allows for the immediate correction of the underlying cause of the arrhythmia. (Choice E) Although atenolol can be used in the management of MAT, initial therapy is directed towards immediate correction of any identified etiology of this tachyarrhythmia. Moreover, this patient's significant smoking history and physical findings (i.e., bilateral wheezing and rhonchi) are suggestive of COPD, which is another cause of MAT; such a condition can be worsened by the use of beta-blockers. Educational Objective: Multifocal atrial tachycardia (MAT) is a tachyarrhythmia that arises in patients with hypoxia, COPD, hypomagnesemia, hypokalemia, heart disease or individuals who are taking aminophylline, theophylline or isoproterenol. It is characterized by the presence of narrow QRS complexes, P waves of 3 or more different morphologies, and variable PR segments and R-R intervals. Therapy is aimed at correcting the underlying cause. 35% of people answered this question correctly; This subscription is licensed to user ID: 123489 only

Item 2 of 2 The patient is admitted to the hospital. His dyspnea has improved, but he continues to have palpitations. His heart rate is 142/min. His blood pressure and respiratory rate remain unchanged. The EKG shows no new changes. His labs reveal the following: CBCHt 50% Platelet count 320,000/cmm Leukocyte count 7,000/cmm Serum chemistrySerum Na Serum K 2.9 mEq/L 125 mEq/L

Chloride 80 mEq/L Bicarbonate 33 mEq/L BUN 25 mg/dL Serum creatinine 1.2 mg/dL Calcium 10.1 mg/dL Blood glucose 84 mg/dL Serum magnesium 2.0 mEq/L Which of the following is the most appropriate pharmacotherapy? B. C. D. E. A. Start intravenous verapamil Start intravenous atenolol Start hypertonic saline infusion Start potassium replacement Prepare for synchronized cardioversion

This subscription is licensed to user: roopika only User ID: 123489 Explanation: The mainstay of therapy of multifocal atrial tachycardia is to treat the underlying cause of the arrhythmia. Hypoxia should be quickly identified and corrected. If the patient fails to improve, electrolyte abnormalities (e.g., hypokalemia or hypomagnesemia) should be sought. This patient has hypokalemia, which could be due to his use of diuretics. He should be treated with potassium replacement. If this measure is not effective, drug therapy is necessary. (Choice A) Verapamil is the preferred calcium-channel blocker for patients with MAT. It should be started only if there is no other correctable cause of MAT. It is effective for rate control, although it converts MATs into sinus rhythms in only 43% of the patients. (Choice B) In the absence of contraindications, beta-blockers such as metoprolol have proven to be effective for the conversion of MATs into sinus rhythms in 80% of the cases; however, atenolol should not be used in this patient because he has bronchospasm and possible COPD. (Choice C) Hyponatremia does not cause MAT; therefore, sodium replacement will not lead to rate control or MAT elimination. His hyponatremia is most likely the result of hydrochlorothiazide. (Choice E) Synchronized cardioversion is not effective in converting MAT into sinus rhythm.

Educational Objective: Treatment of multifocal atrial tachycardia (MAT) is directed towards the correction of the underlying etiology (e.g., hypoxia, hypokalemia, hypomagnesemia). If therapy is not effective and there are no contraindications, beta-blockers can be used successfully. In patients with asthma or COPD, verapamil is the drug of choice. 76% of people answered this question correctly; This subscription is licensed to user ID: 123489 only

A 42-year-old Caucasian woman is brought to the emergency room by paramedics with a change in mental status. She has a past medical history of hypothyroidism, major depression, and chronic back pain. She takes levothyroxine, amitriptyline, and oxycodone regularly. In the emergency room, she is markedly confused, agitated, and keeps talking to herself. On physical examination, her temperature is 38.9 C (102 F), respiratory rate is 22/minute, heart rate is 110/minute, and blood pressure is 92/52 mmHg. There is marked flushing, mydriasis, and hyperreflexia. Her EKG reveals the presence of sinus tachycardia, with widening of the QRS interval. You start her on sodium bicarbonate infusion. While waiting for more data, she has an episode of sustained ventricular tachycardia. Which of the following is the most appropriate antiarrhythmic in this setting? B. C. D. E. A. Lidocaine Procainamide Propranolol Disopyramide Quinidine

This subscription is licensed to user: roopika only User ID: 123489 Explanation: Tricyclic Antidepressants (TCA) are a frequently prescribed group of medications for depression, despite having the highest risk for overdose and suicide attempts. TCA overdose is the leading cause of hospitalization and death. The signs of TCA overdose include hypotension, anticholinergic effects, CNS manifestations, and cardiac arrhythmias. Cardiotoxic effects are responsible for most of the mortality in patients with TCA overdose. TCAs inhibit fast sodium channels, which result to slowing of the phase 0 depolarization in His-Purkinje tissue and the myocardium. This may lead to QRS prolongation and reentrant arrhythmias, like ventricular tachycardia, ventricular fibrillation, and torsades de pointes. Sodium bicarbonate is the most effective agent for the management of TCA-

induced cardiotoxic effects. Lidocaine is the antiarrhythmic drug of choice for TCAinduced ventricular dysrhythmias. (Choice C) Propranolol depresses myocardial contractility and conduction and, thus, enhances the tricyclic toxicity. (Choices B, D, and E) Antiarrhythmic drugs should be used with caution in patients with TCA overdose. Procainamide, disopyramide, and quinidine have membrane-stabilizing effects and enhance TCA toxicity. Educational Objective: Lidocaine is the drug of choice for patients with TCA-induced ventricular arrhythmias. 68% of people answered this question correctly; This subscription is licensed to user ID: 123489 only

A 29-year-old Caucasian male presents to the emergency department with sudden onset chest pain. He says, 'Doc, I have this squeezing in my chest that does not go away.' He has never had such symptoms before. His past medical history is insignificant. He smokes two packs of cigarettes daily, and consumes alcohol occasionally. He uses cocaine and had 'one shot' several hours ago. His family history is significant for a stroke in his father and a heart attack in his mother. His blood pressure is 140/90 mmHg, pulse is 112/min, temperature is 36.7C (98F) and respirations are 24/min. ECG shows 2 mm ST segment elevation in leads I, aVL, V4-V6. The patient receives oxygen per mask, nitroglycerin, aspirin and diazepam. Twenty minutes later, he still complains of chest pain, and his ECG is unchanged. What is the best next step in the management of this patient? B. C. D. E. A. Administer streptokinase Administer phentolamine and observe Administer metoprolol Coronary angiography Reassurance and observation

This subscription is licensed to user: roopika only User ID: 123489 Explanation: Myocardial ischemia is a well-known complication associated with cocaine use. The main factor that leads to myocardial ischemia is vasospasm, although other mechanisms (e.g., increased oxygen demand) also play a role. Cocaine-associated vasospasm may lead to coronary artery thrombosis, which cannot be reversed by vasodilator agents. In patients with transmural ischemia (ST elevation), the initial treatment regimen consists of

nitrates (or calcium channel blockers), aspirin and benzodiazepines. If the patient does not promptly improve with these drugs, immediate coronary angiography should be performed to check for any coronary thrombus. (Choice A) If a coronary thrombus is detected, the next step in management is coronary angioplasty. (Choice B) Phentolamine is a reasonable choice; however, in this case with transmural ischemia, precious time may be lost if phentolamine is given instead of performing coronary angiography, especially if an obstructing thrombus is present. (Choice C) Agents with beta-blocking activity are not recommended because these can exacerbate the condition and may even be associated with worse outcomes. (Choice E) Cocaine-associated myocardial infarction leads to the same complications as 'traditional' myocardial infarction; therefore, a quick and appropriate response to save the jeopardized myocardium is important. Educational Objective: Immediate coronary angiography is performed in patients with cocaine-induced transmural ischemia to check for any coronary thrombus. 31% of people answered this question correctly; This subscription is licensed to user ID: 123489 only

A 38-year-old African-American man is brought to the emergency department (ED) with a two-hour history of sudden onset of sharp, stabbing chest pain under his sternum. He was lifting weights in a health club when the symptoms started. The pain reached its maximum intensity within a few minutes, and is now radiating to his upper back and shoulder tips. His past medical history is not significant. He is a heavy smoker and he uses cocaine frequently. His last cocaine use was six hours ago. His blood pressure is 196/112 mmHg in the right arm and 120/62 mmHg in the left arm. His pulse rate is 120/min and respiratory rate is 18/min. The lung examination is clear to auscultation bilaterally. The cardiovascular examination reveals a regular heart rhythm with an early diastolic murmur heard at the right sternal border. A stat transesophageal echocardiogram is done in the ED, which confirms the diagnosis of acute aortic dissection with extension into the left subclavian artery. What is the best next step in the management of this patient? B. C. D. A. Aspirin Heparin Propranolol Sodium nitroprusside

E.

Hydralazine

This subscription is licensed to user: roopika only User ID: 123489 Explanation: Acute aortic dissection refers to a tear in the aortic intima with separation of the intima from the media, thereby creating a false lumen within the aortic wall. The dissection may propagate either distal or proximal to the intimal tear, causing the associated clinical manifestations. It is usually seen in elderly males with a longstanding history of hypertension and atherosclerosis; however, some conditions associated with or predisposing to aortic dissection in younger patients include a history of connective tissue disorder (Marfan's syndrome, Ehler-Danlos syndrome), inflammatory vasculitis (Takayasu's arteritis, giant cell arteritis, syphilis aortitis), bicuspid aortic valve, coarctation of aorta, use of crack cocaine and trauma. Most of the patients present with a sudden onset of sharp tearing chest or back pain. Patients with ascending aortic dissection may also develop acute aortic insufficiency, thereby causing acute heart failure (with early diastolic decrescendo murmur), acute myocardial infarction due to the dissection extending into coronary vessels, cardiac tamponade or hemothorax or neurological deficits due to the direct extension of dissection into the carotid arteries. Physical examination may reveal a significant difference in the blood pressure between the two arms due to the involvement of the subclavian vessels. The diagnosis is usually suspected based on the history and physical examination, along with mediastinal widening on chest radiograph and the absence of typical electrocardiographic findings of myocardial ischemia or infarction. Transesophageal echocardiography is the modality of choice for rapid confirmation of the diagnosis. Acute aortic dissection involving the ascending aorta is a life-threatening emergency, and all such patients should have emergent surgical intervention. The goal of early medical treatment is to rapidly reduce the shearing stress on the aortic wall and prevent the further propagation of dissection. This is achieved by rapidly lowering the systolic blood pressure and left ventricular contractility with the use of intravenous beta-blockers. Intravenous beta-blockers are the initial drugs of choice in the management of patients with aortic dissection. The goal is to reduce the systolic blood pressure to 100 to 120 mmHg, and to reduce the heart rate to less than 60/min. An intravenous loading dose of propanolol or labetalol followed by an IV infusion can be used to achieve the desired heart rate and blood pressure in most patients. If the blood pressure continues to remain high (systolic blood pressure greater than 100 mmHg), direct vasodilator sodium nitroprusside should be added to the beta-blockers to achieve the desired blood pressure goal. (Choice A) Aspirin has no role in the management of patients with aortic dissection. (Choice B) Heparin is absolutely contraindicated in patients with aortic dissection.

(Choice D) Sodium nitroprusside should only be used in conjunction with beta-blockers. When used alone, it can cause reflex activation of the sympathetic nervous system, thereby causing an increase in the heart rate and ventricular contractility. This, in turn, leads to an increase in the shearing stress on the aortic wall and causes worsening of the aortic dissection. (Choice E) Hydralazine is another direct vasodilator and causes reflex activation of sympathetic nervous system. It has a less predictable blood pressure response and should be avoided in patients with acute aortic dissection. Educational Objective: Intravenous beta-blockers (propranolol or labetalol) should be used in the initial management of patients with acute aortic dissection. Sodium nitroprusside should not be used alone for blood pressure control without prior beta blockade. 37% of people answered this question correctly; This subscription is licensed to user ID: 123489 only

A 66-year-old African-American female comes to the office for a follow-up visit. She was in the hospital two weeks ago with complaints of a sudden onset of dizziness and palpitations. Her initial electrocardiogram in the hospital revealed atrial fibrillation with rapid ventricular response. She was started on diltiazem drip for rate control. The next day, she spontaneously converted back into sinus rhythm. She has a history of hypertension, diabetes mellitus, coronary artery disease status post coronary artery bypass grafting x 2 five years ago, ischemic cardiomyopathy with an ejection fraction of 35%, osteoarthritis, and hypothyroidism. Her medications include daily aspirin, metoprolol, hydrochlorothiazide, lisinopril, atorvastatin, insulin, levothyroxine, warfarin, and ibuprofen as needed. In the office, she tells you that she has been feeling well in general. Occasionally, she has a few episodes of palpitations with slight lightheadedness, which lasts only for a few minutes. Her temperature is 36.7C (98F), blood pressure is 126/82 mmHg, heart rate is 82/min, and respiratory rate is 14/min. Her lungs are clear on auscultation; there are irregularly irregular heart sounds without any audible murmurs. An electrocardiogram done in the office reveals atrial fibrillation. Which antiarrhythmic agent would you use to keep the patient in sinus rhythm? B. C. D. E. A. Ibutilide Diltiazem Verapamil Flecainide Amiodarone

This subscription is licensed to user: roopika only User ID: 123489

Explanation: The patient in the above vignette has a classic presentation of paroxysmal atrial fibrillation. Patients with this condition may present with recurrent episodes of palpitations, which resolve spontaneously over time; however, a great majority of patients may remain asymptomatic and are unaware of the paroxysms of atrial fibrillation. This is especially true if they are on medications to slow their heart rate (betablockers or calcium channel blockers). Patients with paroxysmal atrial fibrillation are at a similar risk of significant thromboembolism and stroke as patients with persistent atrial fibrillation. All patients therefore need to be anticoagulated with warfarin to keep their INR between 2.0 to 3.0; however, there has been a great deal of debate recently regarding the need to keep all such patients in sinus rhythm with the use of antiarrhythmic drugs. In the past, many cardiologists preferred a rhythm control strategy over a rate control one in most patients with atrial fibrillation; however, a number of recent clinical studies have shown that both these approaches are acceptable, and rate control with AV-nodal blocking agents (betablockers, calcium channel blockers, or digoxin) was not inferior to maintenance of sinus rhythm in such patients. Rhythm control strategy is still recommended in patients with marked and persistent symptoms (for example, palpitations, dizziness, dyspnea) or with hemodynamic instability. Recommendations for the use of antiarrhythmic drugs to maintain sinus rhythm are based on the presence or absence of structural heart disease. Structural heart disease is defined by the presence of coronary artery disease, congestive heart failure due to ischemic or non-ischemic cardiomyopathy, or hypertension with left ventricular hypertrophy. In all such patients, amiodarone is the preferred antiarrhythmic drug for the maintenance of sinus rhythm. It has a better efficacy than sotalol and most of the other class I antiarrhythmics and has very low incidence of fatal arrhythmias. (Choice B) Ibutilide is a class III antiarrhythmic drug and has been approved for the acute termination of atrial fibrillation. It is only available in intravenous form and therefore is not useful for long-term maintenance of sinus rhythm. (Choices C and D) Calcium channel blockers (diltiazem or verapamil) have not been shown to prevent recurrence of atrial fibrillation in the long term in patients with paroxysmal atrial fibrillation. (Choice E) Flecainide is a class Ic antiarrhythmic agent and is recommended to maintain normal sinus rhythm in patients without any structural heart disease; however, it can lead to fatal arrhythmias in patients with structural heart disease and can cause a significant increase in mortality. Educational Objective:

Amiodarone is the recommended first line antiarrhythmic drug to maintain normal sinus rhythm in patients with paroxysmal atrial fibrillation and structural heart disease. 45% of people answered this question correctly; This subscription is licensed to user ID: 123489 only

A 62-year-old postmenopausal African-American woman comes to see you in the office. She tells you that she gets a sensation of tightness over her anterior chest after about 1520 minutes of brisk walking. The tightness comes on gradually, and then resolves over the next 5-10 minutes with rest. These episodes are sometimes associated with some difficulty in breathing. The symptoms have been present for the last two months. The patient has a history of hypertension, hyperlipidemia, and a 50-pack-year history of smoking. Her medications include aspirin, metoprolol, hydrochlorothiazide, and simvastatin. She denies any symptoms while she is in the office. Her physical examination, including her vital signs, is unremarkable. The echocardiogram done in the office is also normal. Which of the following is the most appropriate next step in the management of this patient? A. Reassure the patient that she is on adequate treatment for her condition. B. Prescribe nitroglycerin and schedule her for an exercise stress test. C. Admit the patient to the hospital for further diagnostic workup. D. Refer her to a cardiologist for coronary angiography. This subscription is licensed to user: roopika only User ID: 123489 Explanation: The patient in the above vignette has classic symptoms of stable angina pectoris. She has multiple risk factors for coronary artery disease, which are: age greater than 55 years, hypertension, hyperlipidemia, and extensive smoking history. This, along with the typical history of angina pectoris, is highly predictive for the presence of coronary artery disease. All patients with coronary artery disease and stable angina should be referred for an exercise stress test for further risk stratification. Patients with a low risk treadmill score (able to exercise more with a normal EKG) have less than 1% annual mortality rate. On the other hand, patients who have a high-risk treadmill score (presence of chest pain or EKG changes with minimal exercise) have greater than 3% annual mortality rate and require more aggressive management. This patient should therefore be referred for an exercise stress test to obtain further prognostic information.

(Choice C) An exercise stress test for prognostic information can be obtained on an outpatient basis. There is no need to admit the patient to the hospital, unless she is having active chest pain or unstable angina. (Choice D) Coronary angiography is not routinely done in patients with chronic stable angina. The two main indications for coronary angiography are patients with high-risk criteria on exercise stress testing, and patients who continue to have symptoms despite maximal medical therapy. Educational Objective: All patients with chronic stable angina should undergo an exercise stress test for prognostic and risk stratification. 81% of people answered this question correctly; This subscription is licensed to user ID: 123489 only

A 67-year-old Caucasian woman is hospitalized because of dyspnea on exertion and loss of consciousness. Her other medical problems include hypertension and gastroesophageal reflux disease. Her social history and family history are not significant. Her medications include hydrochlorothiazide and famotidine. Work-up showed aortic stenosis, and the patient subsequently underwent aortic valve replacement (AVR) with bileaflet mechanical valve. There is no evidence of left atrial enlargement on the echocardiogram. Her ejection fraction is 50-55%. The preoperative angiogram did not show coronary artery disease. During the entire hospitalization, she is in sinus rhythm. Which of the following is the most appropriate course of action for the long-term management of her condition? B. C. D. E. A. Aspirin 325 mg once daily Warfarin with a goal INR of 1.5-2.5 Warfarin with a goal INR of 2.5-3.5 Warfarin with a goal INR of 4.0-4.5 No need of anticoagulation

This subscription is licensed to user: roopika only User ID: 123489 Explanation: The following are the recommendations from the Sixth ACCP Consensus Conference for a mechanical valve: A goal INR of 2.5 (range 2 to 3) is recommended for patients with AVR with bileaflet mechanical valve, provided that the patient is in sinus rhythm, left atrium is of normal size, and the left ventricular ejection fraction is normal, as in this patient.

A goal INR of 3 (range 2.5 to 3.5) is recommended for patients with MVR with bileaflet mechanical valve or for patients with a bileaflet mechanical aortic valve who have atrial fibrillation. A goal INR of 3 (range 2.5 to 3.5) in combination with low dose aspirin is recommended for patients with mechanical prosthetic valves who suffer a systemic embolism despite adequate anticoagulant therapy. However, more data is coming in favor of a higher level of anticoagulation (target INR >3.0) for both aortic and mitral prosthetic mechanical valves. Educational Objective: The goal INR for mechanical mitral and aortic valves ranges from 2.5 to 3.5. 62% of people answered this question correctly; This subscription is licensed to user ID: 123489 only

A 76-year-old diabetic, African-American man is admitted to the hospital with a threeday history of nausea, vomiting, and generalized abdominal distention. His physical examination and diagnostic workup reveal the presence of a complete small bowel obstruction. He undergoes exploratory laparotomy, and is extubated postoperatively in the surgical intensive care unit without any complications. The day after extubation, he complains of a sudden onset of difficulty in breathing, nausea and marked diaphoresis. On physical examination, his temperature is 36.1C (97F), heart rate is 42/min, respiratory rate is 26/min, and blood pressure is 84/50 mmHg. On lung auscultation, there are bilateral basilar rales present. Cardiovascular examination reveals normal heart sounds with no murmurs or rubs. Chest x-ray is consistent with pulmonary edema. An EKG reveals the presence of marked sinus bradycardia, with 3 mm ST segment elevations in leads II, III, aVF, aVL, V5, and V6, consistent with a diagnosis of acute inferolateral wall myocardial infarction. Which of the following is the most appropriate next step in the management of this patient? B. C. D. E. A. Start the patient on full-dose anticoagulation with heparin. Administer thrombolytic therapy with reteplase. Urgent transvenous cardiac pacing. Start the patient on low-dose dopamine infusion. Administer IV norepinephrine.

This subscription is licensed to user: roopika only User ID: 123489 Explanation:

Sinus bradycardia is commonly seen as a complication of an inferior wall myocardial infarction. It is usually present transiently, immediately after or within the first six hours, and resolves within 24 to 48 hours of acute myocardial infarction. Most of the episodes are asymptomatic, and resolve without any therapy or intervention. Active intervention is required in patients with sinus bradycardia after myocardial infarction, if the patient exhibits signs and symptoms of hemodynamic compromise. The definite treatment is by reperfusion of the infarcted myocardium, either with thrombolysis or percutaneous transluminal coronary angioplasty (PTCA). If bradycardia persists after reperfusion, the patients can still respond to the administration of intravenous atropine. Temporary transvenous cardiac pacing is usually required in patients with persistent bradycardia and hemodynamic instability after myocardial infarction, despite the above measures. This patient is exhibiting the signs of hemodynamic instability with new onset congestive heart failure secondary to sinus bradycardia after an inferolateral wall MI. He should receive a temporary transvenous cardiac pacer until his bradycardia resolves. (Choices A and B) Anticoagulation with full-dose heparin or thrombolysis with reteplase is contraindicated in patients with recent (less than two weeks) intraabdominal surgery. (Choice D) Low-dose dopamine acts predominantly on dopaminergic receptors in the mesenteric, renal and coronary vessels, and causes selective vasodilation. Moderate doses of dopamine stimulate beta 1 adrenergic receptors and increase cardiac output; however, it can worsen the myocardial oxygen demand, and further worsen the hemodynamic status. (Choice E) Norepinephrine acts on both alpha 1 and beta 1 receptors, causing vasoconstriction and an increase in cardiac output. An increase in the blood pressure due to the above effects can cause a reflex bradycardia, thereby causing further decrease in the heart rate. It can also increase the myocardial oxygen demand, and should therefore not be used in patients after an acute myocardial infarction. Educational Objective: Transient sinus bradycardia can occur after an inferior wall myocardial infarction, and is usually asymptomatic. Persistent bradycardia causing hemodynamic instability should be treated with intravenous atropine or temporary transvenous cardiac pacing. 40% of people answered this question correctly; This subscription is licensed to user ID: 123489 only

The following vignette applies to the next 2 items A 28-year-old Mexican-American woman comes to the emergency room because of palpitations. She has no other medical problems. She does not use tobacco, alcohol, or

drugs. She takes no medications. Her temperature is 36.7C (98F), blood pressure is 100/60 mmHg, pulse is 150/min, and respirations are 20/min. A 12-lead EKG is taken, which shows a heart rate of 152 beats/min with no P waves, and with regular and narrowappearing QRS complexes. Item 1 of 2 Which of the following is the most likely diagnosis? B. C. D. E. A. Atrial fibrillation Ventricular tachycardia Sinus tachycardia Supraventricular tachycardia Atrial flutter

This subscription is licensed to user: roopika only User ID: 123489 Explanation: The patient's EKG findings (narrow and regular QRS complexes, absent P wave) are characteristic of a supraventricular tachycardia (SVT). (Choice C) P waves, along with narrow QRS complexes, are present in sinus tachycardia. (Choice A) Atrial fibrillation is an "arrhythmic arrhythmia"; the rhythm is completely irregular, and P waves are absent. (Choice B) Ventricular tachycardia presents with wide QRS complexes (more than 0.12 sec). (Choice E) Atrial flutter is characterized by the presence of "flutter waves" in a saw tooth pattern. Educational Objective: SVT and sinus tachycardia are the most frequent types of tachyarrhythmias. Narrow and regular QRS complexes, and absent P waves suggest supraventricular tachycardia. 52% of people answered this question correctly; This subscription is licensed to user ID: 123489 only

Item 2 of 2

The patient receives antiarrhythmic therapy, and his heart rate decreases from 150/min to 68/min. The palpitations resolve, and the patient begin to feel better. A new 12-lead EKG is taken, which shows a heart rate of 70/min, regular rhythm, presence of P waves, a PR interval of 0.10 sec, a slurred initial part of activation of the QRS complex, a QRS of 0.12 sec, and nonspecific T wave abnormalities. Which of the following is the most likely diagnosis? B. C. D. E. A. Mahaim tachycardia Wolff-Parkinson White (WPW) syndrome AV nodal accessory pathway James fibers preexcitation Right bundle branch block

This subscription is licensed to user: roopika only User ID: 123489 Explanation: The abovementioned features are characteristic of Wolff-Parkinson White (WPW) Syndrome. The slurred upstroke of the QRS complex during the initial part or ventricular activation is known as the delta wave. QRS duration is generally equal to 0.12 sec, but may have greater values. Another feature of WPW syndrome is a PR interval less than 0.12 sec. (Choices A and D) Mahaim and James fibers tachycardias are preexcitation forms that lack delta waves. (Choice E) A right bundle branch block (RBBB) has wide QRS complexes, but does not have delta waves or short PR intervals. Furthermore, an RBBB will not explain the SVT of the patient. (Choice C) AV nodal and AV reentrant tachycardias are the most common etiologies of paroxysmal SVT. Both lack a delta wave and a wide QRS complex, but may have nonspecific T wave abnormalities. Educational Objective: WPW Syndrome can be identified in around 0.2% of the population. It is characterized by a short PR interval (less than 0.12sec), a delta wave at the beginning of the QRS complex, QRS duration of 0.12sec or wider, and non-specific ST segments or T wave abnormalities. Although WPW Syndrome is not common, its diagnosis is important because it can present as SVT. If verapamil or beta-blockers are given, AV nodal conduction will be slow, and the accessory pathway conduction will increase. These can lead to ventricular fibrillation (especially if the patients already have atrial fibrillation) and death. 74% of people answered this question correctly; This subscription is licensed to user ID: 123489 only

A 25-year-old, Caucasian, female athlete comes to see you in the office with complaints of episodes of "a racing heart". The episodes occur almost every day. These last only for a few seconds and occur both during rest and exercise. There is no history of associated dizziness or loss of consciousness with these episodes. She admits to smoking one pack of cigarettes daily, but denies any significant alcohol or illicit drug use. She does not have any other past medical history. Her physical examination reveals the presence of a midsystolic click without any murmurs. An EKG done in the office does not reveal any abnormalities. Holter monitoring reveals the presence of five to seven episodes of nonsustained monomorphic ventricular tachycardia in a day. Which of the following is the most likely cause of the above findings? B. C. D. A. Electrolyte abnormality Congenital long QT syndrome Cocaine abuse Structural heart disease

This subscription is licensed to user: roopika only User ID: 123489 Explanation: Nonsustained ventricular tachycardia is defined as the presence of three or more consecutive ventricular beats with a heart rate greater than 120 beats/min, and with the episode lasting for less than 30 seconds. It is seen most commonly in patients with some form of structural heart disease, possibly because structural abnormalities lead to ventricular arrhythmia. Some structural abnormalities that can lead to nonsustained ventricular tachycardia include scarred myocardium from prior myocardial infarctions (coronary artery disease), ventricular hypertrophy (LVH or hypertrophic obstructive cardiomyopathy), dilated left ventricle (dilated cardiomyopathy), and valvular abnormalities such as mitral valve prolapse. Although nonsustained ventricular tachycardia can also be seen in patients without any evidence of structural heart disease, it is important to recognize that structural heart disease is the most common cause, and a 2D-echocardiogram and stress test should be obtained to rule out ischemia. Based on the Holter monitor findings, the patient's palpitations is due to the presence of nonsustained monomorphic ventricular tachycardia. This is most likely secondary to the presence of structural heart disease, which is most likely due to mitral valve prolapse (midsystolic click). (Choice B) A prolonged QT interval, either due to acquired or congenital QT syndrome, can cause nonsustained polymorphic ventricular tachycardia (also known as torsade de pointes); however, the baseline EKG in the patient did not reveal any evidence of a prolonged QT interval.

(Choice A) Electrolyte abnormalities (hypokalemia or hypomagnesemia) can also cause polymorphic ventricular tachycardia; however, structural heart disease still remains as the most common cause of nonsustained ventricular tachycardia in patients across all age groups. (Choice C) There is no history of cocaine abuse in the patient. Educational Objective: Structural heart disease is the most common cause of nonsustained ventricular tachycardia and should be ruled out before searching for other causes. 69% of people answered this question correctly; This subscription is licensed to user ID: 123489 only

A healthy 27-year-old Caucasian man comes to the physician because of hypertension. He was recently at a community health fair and was told that his blood pressure was 160/90 mmHg. He has no other medical problems. His social and family histories are not significant. He takes no medications. His current blood pressure is 155/95 mmHg. Which of the following physical findings do you expect to find in this patient? B. C. D. E. A. Abdominal bruit Discrepancy between arms and legs blood pressure Retinal abnormalities on funduscopy Edema Tachycardia

This subscription is licensed to user: roopika only User ID: 123489 Explanation: Renovascular disease is the most common cause of secondary hypertension (HTN), and the presence of abdominal or flank bruits is the most common physical finding. Approximately 50% of patients with renovascular hypertension (and 9% of patients with essential hypertension) will have this clinical finding. (Choice C) Retinal abnormalities can be seen in patients with long-standing or uncontrolled hypertension, and are not so common at early stages of the disease. (Choice B) A lower femoral blood pressure, compared to that of the arms, is seen in coarctation of the aorta, which is far less common than renovascular HTN.

(Choice D) Edema can be seen in advanced renal disease and Cushing's syndrome, which are infrequent causes of secondary HTN. (Choice E) Tachycardia is a common finding in pheochromocytoma, which presents in less than 1% of cases of secondary HTN. Educational Objective: Approximately 90% of patients with secondary HTN will have an unidentified cause. Of the remaining, around 3% will be due to renovascular HTN. Other less common causes account for less than 1% each, and are comprised of the following: primary hyperaldosteronism, drug-induced HTN, acute stress, and renal parenchymal disease. Abdominal or flank bruits are present in 50% of the patients with renovascular HTN. This makes this clinical finding the most common and expected clinical finding in a patient with suspected secondary HTN. 48% of people answered this question correctly; This subscription is licensed to user ID: 123489 only

A 50-year-old man comes to the office for a routine physical examination. He complains of several episodes of left-sided, non-radiating chest pain of recent onset. He describes this pain as a slowly-developing, pressure-like sensation which has no aggravating or relieving factors. One week ago, he had an episode of unbearable chest pain, for which he was rushed to the emergency department (ED) with his wife. At that time, he was anxious, diaphoretic, and had shortness of breath. The EKG showed ST changes in the anterior chest leads; however, his cardiac enzymes were normal at that time, so he was discharged from the ED with advice to follow-up with his family physician. He is a construction worker and claims that his job is very hard. He does not exercise. He has had type 2 diabetes mellitus for the past 12 years. His medications include glipizide and multivitamins. Physical examination reveals a temperature of 37C (99F), pulse rate of 80/min, blood pressure of 152/85 mmHg, and respirations of 12/min. His HbA1c is 7% (normal is 4 to 6%). His fasting lipid profile reveals a total cholesterol level of 279 mg/dL and LDL 200 mg/dL. Which of the following is the most important next step in the management of this patient? B. C. D. E. A. Continue current therapy Start him on nitrates Coronary angiography Start gemfibrozil Start thiazide to control blood pressure

This subscription is licensed to user: roopika only User ID: 123489 Explanation:

This patient has unstable angina, and he needs coronary angiography as soon as possible to diagnose the degree and location of blockage in the coronary circulation. He has several major risk factors for coronary artery disease, including diabetes mellitus, hypertension, and hyperlipidemia. Coronary angiography will be extremely useful in deciding the final management of this patient. The patient with a high risk of an acute coronary event should be treated aggressively by performing coronary angiography, followed by percutaneous coronary intervention (PCI) or CABG. In patients with diabetes, the coronary artery disease is usually severe and progresses rapidly. (Choice A) This patient is not doing well with his current therapy; therefore, continuing with his therapy could be disastrous. (Choice B) Nitrates would be helpful in alleviating his chest pain, but will not affect the ultimate survival of this patient. The long-term use of nitrate in preventing acute coronary events or coronary deaths is unclear. (Choice D) Gemfibrozil mainly reduces the triglyceride levels, and is the first line agent in patients with primary hypertriglyceridemia. (Choice E) Thiazide is not a good option for this patient with unstable angina. Other antihypertensive drugs which have been shown to improve the survival of patients with CAD (i.e., beta-blockers, ACEI) can be more beneficial. Educational Objective: Patients with unstable angina and with high risk for an acute coronary event should be managed with early coronary angiography, followed by PCI or CABG. Patients with diabetes usually have extensive coronary artery disease, which progresses rapidly. 54% of people answered this question correctly; This subscription is licensed to user ID: 123489 only

An 82-year-old Caucasian man presents to the emergency department for the evaluation of increased urinary frequency and burning, and mental confusion. He is admitted to the hospital, and is started on intravenous fluids and antibiotics. The next day, during the rounds, he is found to be in mild respiratory distress. His physical examination reveals the presence of jugular venous distention (9 cm) and bilateral basilar crackles. His chest x-ray is consistent with pulmonary edema. A 2D echocardiogram done two months ago revealed an ejection fraction of approximately 35%. You give the patient 40 mg of IV furosemide (Lasix), after which his respiratory status improves. His mental status remains unchanged, and he occasionally has episodes of agitation and visual hallucinations. The attending physician tells you to start him on medications for heart failure that would not

affect his mental status. Which of the following is the most appropriate drug to start at this point? B. C. D. A. Lisinopril Digoxin Spironolactone Verapamil

This subscription is licensed to user: roopika only User ID: 123489 Explanation: The patient in the above vignette is admitted to the hospital with urinary tract infection and possible urosepsis. Acute infections are a common cause of delirium, especially in elderly patients being admitted to the hospital. An important goal of treatment in such patients is to avoid the use of medications that can potentially worsen the acute delirious state. Diuretics, ACE inhibitors, spironolactone, beta-blockers, and digoxin are all a part of standard therapy for patients with heart failure or cardiomyopathy due to any cause. ACE inhibitors (lisinopril) have consistently been shown in multiple, large, randomized trials to have a significant beneficial effect on patients with heart failure. They have a very low incidence of central nervous system (CNS) side effects, and do not cause worsening of confusional states or delirium. Lisinopril can be safely started in this patient without the risk of exacerbating his confusion and agitation. (Choice B) Digoxin is a cardiac glycoside frequently used in patients with congestive heart failure due to systolic dysfunction and atrial dysrhythmias; however, its use has been associated with significant CNS side effects. These include blurred vision, dizziness, confusion, mental disturbances, anxiety, delirium, and hallucinations. Its use should be avoided in patients with ongoing acute mental confusion or delirium. (Choice C) Spironolactone is a useful adjunct in the management of patients with congestive heart failure, especially in patients with a low ejection fraction; however, its use can also cause CNS disturbances such as drowsiness, lethargy, and mental confusion. (Choice D) Verapamil is a calcium channel blocker used in the treatment of hypertension, angina, and supraventricular tachyarrhythmias. It has a lower incidence of CNS side effects, but can precipitate or exacerbate heart failure symptoms. It should be avoided in patients with congestive heart failure. Educational Objective: Digoxin use can exacerbate confusion and mental disturbances and should be avoided in patients with delirium. 61% of people answered this question correctly; This subscription is licensed to user ID: 123489 only

A 63-year-old Caucasian man comes to the office for a routine follow-up visit. He has bilateral knee osteoarthritis that is well controlled with naproxen. He does not use tobacco, alcohol or drugs. He takes no other medications. Examination reveals a 2/6 ejection systolic murmur at right 2nd intercostal space. The patient is referred for a 2Dechocardiogram, which revealed: mild aortic sclerosis and left ventricular dilatation with an ejection fraction of 40%. Which of the following is the most effective pharmacotherapy? B. C. D. E. A. Isosorbide dinitrate or mononitrate Carvedilol Enalapril Low-sodium diet Diuretics

This subscription is licensed to user: roopika only User ID: 123489 Explanation: ACE inhibitors (i.e. enalapril, ramipril, captopril and lisinopril) have been proven to be beneficial in the treatment of heart failure (CHF). These medications improve survival, and delay progression of the disease. They must be started at low doses, and titration should be done as tolerated. ACE inhibitors are especially effective in men and the Caucasian population. Although there is currently insufficient data to assess the benefit of ACE inhibitor therapy to the mortality of women, these drugs are still used in the treatment of CHF in this group. This patient has asymptomatic left ventricular dysfunction and aortic sclerosis; however, the former cannot be attributed to the latter. Asymptomatic aortic sclerosis without stenosis is common in elderly patients. It is usually detected as an ejection systolic murmur on physical examination, or by echocardiography. This may progress to aortic stenosis over a period of time. ACE inhibitors are still indicated for the management of the patient's heart failure, even if he has no symptoms. (Choices D and E) A low-sodium diet and diuretics are indicated in patients with some degree of fluid retention and hypertension, not for asymptomatic patients. (Choice B) Beta-blockers are not indicated in the asymptomatic phase of HF. (Choice A) Isosorbide mononitrate is a long-acting vasodilator used in angina pectoris management. Isosorbide dinitrate is used to treat acute pulmonary edema in HF. Educational Objective:

ACE inhibitors are the main therapy for CHF. They are indicated even in the asymptomatic phase. Although there is currently insufficient data to show its benefits to the mortality rate of African-Americans and women, their use in CHF is always recommended. The only reasons for not giving ACE inhibitors are (1) poor tolerance to the drugs (presence of severe adverse effects) and (2) the presence of contraindications, such as renal failure (Creatinine greater than or equal to 3 mg/dL) or hyperkalemia. 51% of people answered this question correctly; This subscription is licensed to user ID: 123489 only

A 70-year-old Caucasian man is brought to the emergency room by his wife after an episode of syncope while working in his garden. His wife says that he has always been physically active, but complained of increased fatigability lately. His past medical history is significant for diabetes mellitus type-2 and gout. His only current medication is glyburide. His blood pressure is 100/80 mmHg, and heart rate is 90/min. Physical examination is significant for a harsh crescendo-decrescendo type murmur that is heard at the base of the heart and radiates to the carotids. ECG reveals left ventricular hypertrophy and secondary ST segment and T wave changes. Echocardiography shows concentric left ventricular hypertrophy with severe calcification of the aortic valve. The aortic valve area is 0.78 cm2, and the trans-valvular gradient is 50 mmHg. Which of the following is the best statement about this patient's condition? A. prognosis. B. C. D. E. The onset of symptoms minimally, if at all, affects the

Conservative treatment should be tried before offering an intervention. Balloon valvulotomy is preferred due to low procedural morbidity. Balloon valvulotomy is preferred due to better long-term prognosis. Aortic valve replacement is associated with marked reduction in mortality.

This subscription is licensed to user: roopika only User ID: 123489 Explanation: This patient presents with symptomatic aortic stenosis. An aortic valve area of less than 1.0 cm2 is considered severe stenosis. The onset of symptoms markedly affects the prognosis in patients with aortic stenosis (Choice A); therefore, prompt intervention is recommended in patients who have cardinal symptoms of aortic stenosis (dyspnea of heart failure, anginal pain, and syncope). Aortic valve replacement is associated with marked reduction in symptoms and mortality in patients with symptomatic aortic stenosis, and is considered the treatment of choice. (Choices C and D) Balloon valvulotomy is associated with high procedural morbidity and transient efficacy. It is considered only in selected clinical settings (e.g. patients with hemodynamic instability or significant comorbidities).

(Choice B) Conservative treatment has little role in the management of patients with aortic stenosis, because most medical interventions can destabilize the patients (e.g. diuretics or vasodilators). Educational Objective: Aortic valve replacement is associated with marked reduction in symptoms and mortality in patients with symptomatic aortic stenosis, and is considered the treatment of choice. 62% of people answered this question correctly; This subscription is licensed to user ID: 123489 only

The following vignette applies to the next 3 items A 70-year-old African-American woman is brought to the emergency department because she woke up this morning with a sudden onset of shortness of breath. She has never had angina or shortness of breath. She denies wheezing, chest pain, leg swelling, palpitations, and loss of consciousness. Her other medical problems include hypertension, diabetes mellitus-type 2, and hypercholesterolemia. She had bronchial asthma when she was a child. Her only hospital admission occurred two months ago, when she underwent a spinal fusion for lower back problems. She denies the use of alcohol or tobacco. Her family history is not significant. Her temperature is 36.1 C (97 F), blood pressure is 210/106 mmHg, pulse is 65/min, and respirations are 34/min. Her pulse oximetry reading is 81% at room air. Physical examination shows diffuse crackles all over the lung fields. Her labs reveal the following: CBCHb 11.4 g/dL Platelet count 180,000/cmm Leukocyte count 8,000/cmm Her chest x-ray is shown below. Oxygen is administered.

Item 1 of 3 Which of the following is the most appropriate next step in the management of this patient? B. C. D. A. Give IV nitroglycerine Give intravenous metoprolol Administer furosemide Start intravenous heparin

E.

Albuterol nebulization

This subscription is licensed to user: roopika only User ID: 123489 Explanation: The patient's chest x-ray is characteristic of flash pulmonary edema, which is most likely due to hypertensive crisis. The initial steps in management of patients with acute pulmonary edema includes the administration of oxygen, morphine, and loop diuretics (IV furosemide). Educational Objective: Patients with acute pulmonary edema are initially managed with oxygen, morphine and loop diuretics (IV furosemide). 61% of people answered this question correctly; This subscription is licensed to user ID: 123489 only

Item 2 of 3 The patient is given the appropriate treatment. She also undergoes further evaluation. Her EKG reveals left ventricular hypertrophy with repolarization changes. The first set of cardiac enzymes is negative. She is still in moderate distress, and has similar vital signs. What is the next best step in the management of this patient? B. C. D. E. A. Add intravenous steroids Start dopamine Increase metoprolol Start IV nitroglycerine Continue heparin

This subscription is licensed to user: roopika only User ID: 123489 Explanation: This patient's flash pulmonary edema is most likely due to hypertensive crisis. The two most commonly used antihypertensive agents in the management of hypertensive crisis complicated by acute pulmonary edema are IV nitroglycerine and IV nitroprusside. (Choice B) Dopamine is indicated in patients with cardiogenic shock complicated by hypotension. This patient has hypertensive crisis; therefore, the use of dopamine in this case is very inappropriate.

(Choice E) Heparin is indicated in either pulmonary embolism (PE) or acute coronary event; however, there is no evidence of ischemia (normal EKG), no chest pain, and no elevated cardiac enzymes, which could suggest the latter. PE is also unlikely in this case, given the patient's findings of pulmonary edema in the chest x-ray, as well as hypertension. (Choice C) Beta-blockers are not usually given in acute pulmonary edema. (Choice A) Steroids are indicated in acute bronchial asthma or COPD exacerbation, which is not the case here. Educational Objective: The two most commonly used antihypertensive agents in the management of hypertensive crisis complicated by acute pulmonary edema are IV nitroglycerine and IV nitroprusside. 58% of people answered this question correctly; This subscription is licensed to user ID: 123489 only

Item 3 of 3 The patient is responding well to the treatment. The second set of cardiac enzymes is negative. What other test is necessary for this patient? B. C. D. E. A. Echocardiogram Coronary angiogram Pulmonary function tests V/Q scan and D-dimer Renal duplex scan

This subscription is licensed to user: roopika only User ID: 123489 Explanation: Mitral stenosis and acute aortic or mitral regurgiation can sometimes present with flash pulmonary edema; therefore, an echocardiogram should be performed to evaluate the systolic and diastolic function, and to check for any left ventricular hypertrophy or valvular disease. (Choice E) A renal duplex scan is indicated in patients with severe, uncontrolled hypertension and recurrent episodes of flash pulmonary edema. Educational Objective:

All patients with flash pulmonary edema of unknown etiology should be evaluated with an echocardiogram. 65% of people answered this question correctly; This subscription is licensed to user ID: 123489 only

A 68-year-old Caucasian woman comes to your office for follow-up of her anticoagulation therapy. She has chronic atrial fibrillation, hypertension, congestive heart failure NYHA II, and coronary artery disease. She takes warfarin (Coumadin, 6 mg), lisinopril, and carvedilol daily. Her last echocardiogram performed two months ago shows a dilated left atrium measuring 58 mm. Her last laboratory evaluation, done early in the morning, shows an INR of 4.8 with a prothrombin time of 20 sec. She denies any kind of bleeding episodes. Which of the following measures is the most appropriate to manage her situation? B. C. D. E. A. Give 1-5 mg of vitamin K per mouth. Give 1-5 mg of subcutaneous vitamin K. Transfuse 1-2 units of Fresh Frozen Plasma (FFP). Hold the Coumadin for 1-2 days. Give 5- 10 mg of intravenous vitamin K.

This subscription is licensed to user: roopika only User ID: 123489 Explanation: Prospective studies have shown that if the INR is less than 5, holding Coumadin for one day or decreasing its dose will make the INR fall to its therapeutic dose. (Choices C and E) Fresh frozen plasma, intravenous vitamin K, or prothrombin complex concentrates are only needed if the patient is bleeding or if the INR is greater than 20. In this case, the use of FFP or intravenous vitamin K can completely reverse anticoagulation and pose the peril of thrombotic events especially since this patient has CHF, atrial fibrillation, and a dilated atrium. Furthermore, the use of intravenous vitamin K can lead to the development of a temporary state of Coumadin resistance, which may increase the odds of thrombosis by prolonging the subtherapeutic anticoagulation period. (Choice A) If the patient is not bleeding and her INR is between 5 and 9, Coumadin should be withheld and a small dose (1-2.5 mg) of vitamin K can be added. If the INR is greater than 9 but lower than 20, higher doses (3-5 mg) of oral vitamin K are usually needed. (Choice B) Subcutaneous vitamin K is only used when oral vitamin K is not available because it has a slower onset of action and no added benefit.

Educational Objective: The use of FFP or intravenous vitamin K is only necessary if the patient is bleeding or has an INR greater than 20. If the INR is supratherapeutic, but not higher than 5, it is enough to hold the Coumadin or decrease its dose. All other cases of excessive anticoagulation with Coumadin can be successfully managed with oral vitamin K. 66% of people answered this question correctly; This subscription is licensed to user ID: 123489 only

A 55-year-old Caucasian man comes to the physician because of exertional substernal chest pain radiating to the left arm. The pain is usually relieved within 5 to 10 minutes of rest. He has had these symptoms for the past two months. He denies shortness of breath, palpitations, and loss of consciousness. His other medical problems include hypertension and diabetes mellitus-type 2. He has smoked two packs of cigarettes daily for 24 years. He drinks 1-2 beers daily. His family history is not significant. His medications include metformin and ramipril. His temperature is 36.1 C (97 F), blood pressure is 140/80 mmHg, pulse is 78/min, and respirations are 16/min. Examination shows no abnormalities. Which of the following is the most appropriate next step in the management of this patient? B. C. D. E. F. A. Schedule outpatient exercise stress testing. Schedule outpatient pharmacological stress testing. Refer to cardiology for coronary angiography. Schedule 2D-echocardiography. Obtain an electrocardiogram in the office. Advise him to stop smoking and schedule follow-up visit.

This subscription is licensed to user: roopika only User ID: 123489 Explanation: Exercise stress testing is useful for the diagnosis and risk stratification of patients with stable angina. Exercise testing is useful to identify a high-risk group that will benefit from coronary angiography. Patients at high risk have one of the following findings on exercise testing: 1. failure to increase their blood pressure with exercise 2. inability to complete stage I of Bruce protocol 3. appearance of horizontal or downsloping ST segment during exercise This patient does not need to be admitted in the hospital at this point. The workup can be done in an outpatient setting.

(Choice B) Pharmacological stress testing is an alternative to exercise stress testing, and is indicated in the setting of CAD, when the patient is unable to exercise due to some underlying medical disorder (amputation), when an exercise stress test is relatively contraindicated (MI, unstable angina), or in patients in whom interpretation of an EKG is difficult because of preexisting changes, such as left bundle branch block, baseline ST/T changes from left ventricular hypertrophy, etc. (Choice C) Coronary angiography is indicated in the setting of stable angina pectoris when the angina is refractory to medical treatment, or when exercise testing identifies the patient as high-risk. (Choices D and E) Exercise echocardiography is more sensitive than the resting echocardiogram or stress EKG, and is also commonly done in patients with stable angina. Resting echocardiography or EKG is useless in stable angina when the patient has no active chest pain. (Choice F) Even though advising risk factor modification is extremely important, this should not be the only action taken in this patient's management. He should have a stress test first, and, if positive, cardiac catheterization. Educational Objective: A stress EKG or an exercise echocardiogram should be considered for risk stratification in patients with stable angina. 36% of people answered this question correctly; This subscription is licensed to user ID: 123489 only

A 53-year-old Caucasian businessman is brought in to the emergency department by coworkers. He was in a meeting at work when he suddenly experienced severe pain and tightness in his chest. The pain radiates to his jaw. He reports feeling very anxious and lightheaded, and is extremely sweaty upon arrival. He was given supplemental oxygen and intravenous access was established. His vital signs include temperature of 36.9C (98F), blood pressure of 132/90 mm Hg, pulse of 82/min, and respirations are 20/min. No electrocardiogram results are yet available. What medication should be administered next? B. C. D. E. A. Aspirin Captopril Heparin Metoprolol Streptokinase

This subscription is licensed to user: roopika only User ID: 123489

Explanation: Unless there is clear evidence to the contrary, clinicians managing chest pain should always assume it is ischemic in origin. Supplemental oxygen should be administered, pulse oximetry readings obtained, and intravenous access established. Ideally, 325 mg of aspirin (Choice A) should be chewed en route to the hospital. Nitroglycerin is typically given next, unless contraindications exist (e.g., hypotension or recent usage of sildenafil). ACE inhibitors such as captopril (Choice B) are not as commonly given in the acute setting. However, they are known to significantly improve cardiac hemodynamics and reduce mortality when given for a several week period following myocardial infarction. Heparin (Choice C) may be given intravenously after aspirin, nitroglycerin, and morphine have been administered and the electrocardiogram performed. Beta-blockers such as metoprolol (Choice D) are of help in controlling heart rate and decreasing myocardial oxygen demand, and are typically given after aspirin, nitroglycerin, and morphine have been administered. Streptokinase (Choice E) is used for thrombolysis as an alternative to primary coronary intervention (e.g., stent placement or balloon dilation), but is not one of the first medications administered after myocardial infarction. Educational Objective: In patients whose chest pain may be ischemic in origin, supplemental oxygen and aspirin should be administered immediately, followed by nitroglycerin, morphine, and betablockers (presuming no contraindications exist). 91% of people answered this question correctly; This subscription is licensed to user ID: 123489 only

A 62-year-old Caucasian man comes to the office for the evaluation of progressive shortness of breath for the last two months. He used to walk two miles a day without any difficulty. He has recently noticed that he gets "out of breath" and tires easily after walking a few blocks or even after walking upstairs. He denies any history of fever, chills, cough, weight loss, or night sweats. Which of the following is the best initial method to diagnose congestive heart failure? B. C. A. History and physical examination Electrocardiogram Chest radiograph

D. E.

Plasma B-type natriuretic peptide 2D-echocardiogram

This subscription is licensed to user: roopika only User ID: 123489 Explanation: Congestive heart failure is a clinical syndrome that results from impaired or inadequate ventricular emptying (systolic dysfunction) or impaired ventricular relaxation (diastolic dysfunction). It is characterized by specific symptoms that are either due to reduced cardiac output (fatigue, weakness) or excessive fluid retention (peripheral or pedal edema, dyspnea). All symptoms are exacerbated by exertion, and the degree of these symptoms helps in assessing the severity of heart failure. Heart failure is primarily a clinical diagnosis and is based on a detailed history and physical examination. A detailed history and physical examination can also provide clues to the specific cause of heart failure. The major criteria include the presence of paroxysmal nocturnal dyspnea, orthopnea, raised jugular venous pressure, pulmonary rales, presence of third heart sound, increased cardiac silhouette, and pulmonary vascular congestion on chest x-ray. The minor criteria include the presence of bilateral lower extremity edema, nocturnal cough, dyspnea on exertion, tachycardia, presence of pleural effusion, and hepatomegaly. The diagnosis of heart failure is based on the presence of two major criteria or one major and two minor criteria, provided the minor criteria cannot be explained by the presence of any concurrent medical illness. (Choice B) An electrocardiogram may provide clues to the specific cause of heart failure, but is not helpful for the initial diagnosis of congestive heart failure. (Choice C) Chest radiograph findings suggestive of congestive heart failure include cardiomegaly, pulmonary vascular congestion (especially in the upper lobes), Kerley B lines and bilateral pleural effusions. The presence or absence of such findings will help to differentiate heart failure from dyspnea secondary to a primary pulmonary pathology. (Choice D) Plasma B type natriuretic peptide levels are useful at times to differentiate between cardiogenic versus non-cardiogenic causes of dyspnea; however, the history and physical examination are primarily used to diagnose congestive heart failure initially. (Choice E) 2D-echocardiogram is very helpful in confirming the diagnosis, and also provides clues to the cause and severity of heart failure; however, a history and physical examination should be initially performed in all patients prior to obtaining the 2Dechocardiogram. Educational Objective: Obtaining a detailed history and physical examination is the initial method of choice to diagnose congestive heart failure prior to obtaining any confirmatory diagnostic tests.

75% of people answered this question correctly; This subscription is licensed to user ID: 123489 only

The following vignette applies to the next 2 items A 55-year-old Caucasian man comes to the emergency department with a sudden onset of retrosternal chest pain that began eight hours ago. An electrocardiogram revealed the presence of ST segment elevation in leads V2 to V5. He is taken emergently for cardiac catheterization, where he undergoes percutaneous transluminal coronary angioplasty with stent placement to the left anterior descending artery. After the procedure, the ST changes resolve, and the patient has an uneventful hospital course. On the third night, just prior to his discharge, he called for the nurse due to a sudden onset of sharp, retrosternal chest pain. He first noticed the pain while he was turning around in the bed. The pain gets worse with deep breathing. His vital signs are as follows: temperature 37.8C (100F), blood pressure 134/80 mmHg, heart rate 108/min, and respiratory rate 22/min. Physical examination reveals jugular venous distention (3 cm), regular heart sounds without any murmurs, and clear lung fields. There is a scratchy sound heard during ventricular systole over the left sternal border. An EKG done during the episode reveals the presence of sinus tachycardia with Q waves in leads V2 to V5. Item 1 of 2 Which of the following is the most likely cause of this patient's recurrent symptoms? B. C. D. E. A. Acute pericarditis Left ventricular aneurysm Chordae tendineae rupture Acute myocardial infarction Dressler's syndrome

This subscription is licensed to user: roopika only User ID: 123489 Explanation: The patient has a clinical syndrome consistent with a diagnosis of acute pericarditis. Acute pericarditis (infarct associated or infarction pericarditis) can occur within one to four days as a direct complication of a transmural myocardial infarction. The recurrence of chest pain three days after a myocardial infarction, which gets worse with position changes and deep inspiration, is suggestive of infarction pericarditis. It is usually a clinical diagnosis, and is supported by the presence of pericardial friction rub and EKG changes of pericarditis (sinus tachycardia, diffuse ST segment elevations with PR segment depression). A pericardial rub is usually heard over the left sternal border and

can be present during any of the phases of the cardiac cycle. It is heard as a superficial scratchy or grating sound, which gets more pronounced when the patient leans forward. The EKG changes of pericarditis are not always seen, but usually resemble that of an acute myocardial infarction. Infarction pericarditis is seen less frequently in patients with early and complete reperfusion. It is usually a transient episode and does not affect the management of acute myocardial infarction, unless it is complicated by a large pericardial effusion or tamponade. (Choice B) A left ventricular aneurysm usually occurs as a late complication of a transmural myocardial infarction (usually a large anterior wall MI). It may result in heart failure, ventricular arrhythmias, or peripheral arterial embolization due to the formation of a left ventricular thrombus. It develops over a longer period of time, and is usually not associated with chest pain. Persistent ST elevation can be present in these patients. (Choice C) Chordae tendineae or papillary muscle rupture is a life threatening mechanical complication of an acute myocardial infarction. It usually occurs two to seven days after the infarction and causes acute hemodynamic instability. Patients also develop acute pulmonary edema. (Choice D) Acute myocardial infarction is unlikely in the absence of any specific EKG changes of ischemia or infarction. The presence of 'Q' waves indicates an old infarction in this patient. (Choice E) Dressler's syndrome, or post-cardiac injury syndrome, occurs in patients with myocardial infarction and after cardiac surgery. It is an autoimmune mediated syndrome, which usually develops weeks to months after an acute MI. It usually presents with fever, leukocytosis, pleuritic chest pain, and a pericardial rub. Educational Objective: Acute pericarditis (infarction pericarditis) can occur within one to four days as a complication of transmural myocardial infarction. 48% of people answered this question correctly; This subscription is licensed to user ID: 123489 only

Item 2 of 2 Which of the following is the most appropriate next step in the management of this patient? A. NSAIDs

B. C. D. E.

Glycoprotein IIb/IIIa inhibitors Oral corticosteroids Urgent cardiac catheterization for in-stent restenosis Immediate cardiothoracic surgical referral

This subscription is licensed to user: roopika only User ID: 123489 Explanation: Acute pericarditis occurring after a myocardial infarction is usually transient (lasts for only a few days). It should be managed with close clinical observation and adequate pain control. Nonsteroidal anti-inflammatory medications (NSAIDs) are effective in alleviating the pain associated with acute pericardial inflammation, although there are some concerns that they may increase the risk of myocardial rupture after a transmural MI. Of all the given options, NSAIDs is the treatment of choice for controlling the symptoms in patients with acute pericarditis. (Choices B and D) There is no evidence of myocardial ischemia or infarction in this patient. Glycoprotein IIb/IIIa inhibitors and emergent cardiac catheterization are therefore not indicated in this patient. (Choice C) Corticosteroids are useful in refractory cases of Dressler's syndrome, when nonsteroidal antiinflammatory drugs fail to control the symptoms. Its use has been reported to increase the incidence of ventricular aneurysm formation. (Choice E) The patient in the above vignette does not exhibit any signs of acute mitral regurgitation (MR) or chordae tendinae rupture. Urgent cardiothoracic surgical evaluation and management are required for patients with chordae tendineae rupture since it leads to an acute MR and hemodynamic instability. Educational Objective: Acute pericarditis following a transmural myocardial infarction is usually transient and should be managed by pain control with NSAIDs. 71% of people answered this question correctly; This subscription is licensed to user ID: 123489 only

A 30-year-old male comes to your office because he was referred to you for further evaluation of his hypertension. He is completely asymptomatic. He usually runs 2-3 miles every day without any discomfort. His blood pressure is 150/90 mmHg, and pulse rate is 52/min, regular. On auscultation, S1 and S2 are normal. There is a II/VI systolic ejection murmur heard at the left lower sternal border and radiates to the apex. There is no carotid upstroke delay; however, there is a delay in the femoral pulse. There is no elevation of

JVD or any pedal edema. His EKG showed left ventricular hypertrophy. The twodimensional echocardiogram revealed a normal left ventricular cavity size with an ejection fraction of 65%. There is no left ventricular hypertrophy, but there is evidence of a bicuspid aortic valve. Based on the history and physical findings, what is the most appropriate next step in diagnosing this patient's condition? A. Coronary angiogram with left and right heart catheterization B. CT of the chest C. MRI of the chest D. Transesophageal echocardiogram E. Exercise treadmill test This subscription is licensed to user: roopika only User ID: 123489 Explanation: This patient has classic signs and symptoms of coarctation of the aorta (COA). This is usually associated with a congenital bicuspid aortic wall. In a previously undiagnosed patient, the classic presenting sign is hypertension. Most patients are asymptomatic unless severe hypertension is present, which can lead to headaches, epistaxis, heart failure, or dissecting aneurysm. The major clinical manifestation in children and adults with COA is a difference in the systolic blood pressure between the upper and lower extremities. In most cases, the diastolic blood pressures are similar. The classic findings are hypertension in the upper extremities, diminished or delayed femoral pulses, and lower unobtainable arterial blood pressures in the lower extremities. Magnetic resonance imaging (MRI) is the best noninvasive modality to diagnose COA because it can usually clearly define the location of severe coarctation of the aorta. MRI can also detect associated cardiac abnormalities and can be used for serial follow-up after surgical repair or balloon angioplasty. (Choice B) In the presence of contraindications to MRI, computerized tomographic imaging can be used to diagnose COA. (Choice D) Transesophageal echocardiogram (TEE) is an invasive method that is usually performed under light sedation. As a result, TEE is not used to make a diagnosis of COA, especially since transthoracic echocardiography can provide excellent images of the aortic arch when obtained from the substernal notch. (Choice A) Given the accuracy of noninvasive methods for diagnosis, a major indication for cardiac catheterization for isolated COA is the planning of a therapeutic intervention. Catheterization may also be necessary when coarctation is associated with complex cardiac defects. Educational Objective:

Recognize the clinical features of coarctation of the aorta (COA). MRI is the best noninvasive modality used to diagnose COA. 8% of people answered this question correctly; This subscription is licensed to user ID: 123489 only

A 56-year-old Caucasian man is brought to the emergency department with a four hour history of crushing, substernal chest pain. His electrocardiogram shows a 3 mm ST segment elevation in leads V2 - V4. He is taken straight for cardiac catheterization and has a primary angioplasty and stenting of the left anterior descending artery. After the procedure, he is admitted to the coronary care unit. He recovers slowly over the next four days, and begins to participate in a cardiac rehabilitation program. Throughout his hospital stay, he remains asymptomatic. On the day of his discharge, he asks you whether it would be safe for him to resume sexual activity. Which of the following is the most appropriate response in this setting? A. It is safe to resume sexual activity after discharge B. It is safe to resume sexual activity after 1 week C. It is safe to resume sexual activity after 6 weeks D. It is safe to resume sexual activity after discharge as long as he is asymptomatic This subscription is licensed to user: roopika only User ID: 123489 Explanation: An important issue for many patients after a myocardial infarction is when sexual activity can be safely resumed; therefore, this should be addressed in all sexually active patients before hospital discharge. A consensus panel concluded that patients resuming sexual activity six weeks after an uncomplicated myocardial infarction are at low risk for future myocardial events. On the other hand, those resuming sexual activity within six weeks are at intermediate risk, and those resuming activity within two weeks are at high risk. Patients with complications such as post-infarction chest pain, evidence of arrhythmias, or heart failure are also at intermediate or high risk. Patients at intermediate risk should receive further evaluation and subsequent reclassification into the low-risk or high-risk category, while patients at high risk should be stabilized with appropriate therapy before resuming sexual activity. Consultation with a cardiologist prior to resuming sexual activity is recommended in all high-risk patients. Educational Objective:

Patients who resume sexual activity six weeks after an uncomplicated myocardial infarction are at low risk for future myocardial events. Patients with complications should be further evaluated before they can safely resume sexual activity. 53% of people answered this question correctly; This subscription is licensed to user ID: 123489 only

A 55-year-old male with type 2 diabetes and hypercholesterolemia is brought to the emergency department (ED) because of an acute, substernal chest pain. His chest pain resolves after taking sublingual nitroglycerine and aspirin. The patient uses dietary means to control his blood glucose levels, which are currently borderline. He takes 20 mg of simvastatin every day. The initial electrocardiogram shows a new 1-mm ST segment depression in leads II, III, aVF, V2-V5. After admission, his cardiac enzymes are found to be elevated. Cardiac catheterization shows >90% blockage in the right coronary, left circumflex and left anterior descending arteries. His HBA1c is 7.2 % (normal 4 - 6 %) and his current LDL is 120 mg/dl (goal <100 mg/dl in diabetes). What is the most effective way of dealing with this patient's coronary artery disease? B. C. D. E. A. Start insulin therapy Coronary artery bypass graft Angioplasty with stent placement Balloon angioplasty alone Increase the dose of simvastatin

This subscription is licensed to user: roopika only User ID: 123489 Explanation: Studies have shown that compared to angioplasty, coronary artery bypass graft (CABG) improves long-term survival in diabetic patients with multivessel disease and recent Qwave infarction. Good, perioperative control of the blood glucose levels is advocated during CABG. An insulin drip achieves better blood glucose control during CABG, and decreases the incidence of sternotomy wound infections. (Choices C and D) Aside from the abovementioned reasons, trials have shown that patients with diabetes have a higher incidence of restenosis after angioplasty. (Choice A) Insulin treatment could be necessary in this patient during cardiac surgery. Furthermore, improved glycemic control after an acute myocardial infarction has been shown to improve long-term survival; however, giving insulin treatment without subjecting the patient to CABG would be detrimental.

(Choice E) Similarly, lowering of LDL is essential, but if done alone, would not be enough for opening up the significant blockage in the patient's coronary arteries. Improved glycemic control and lower LDL levels are likely to be beneficial in the long run. Educational Objective: Coronary artery bypass graft is indicated in patient's with severe 3-vessel coronary artery disease or left main disease. 80% of people answered this question correctly; This subscription is licensed to user ID: 123489 only

A 62-year-old Caucasian man comes to the physician's office with complaints of intermittent chest pain and shortness of breath for the past two months. His symptoms are worsened by exertion and relieved by rest. His other medical problems include hypertension and gastroesophageal reflux disease. His medications are hydrochlorothiazide, enalapril, and ranitidine (as needed, for reflux symptoms). He works as a paramedic. He has smoked one pack of cigarettes daily for 35 years, and drinks 1-2 bottles of beer daily. His physical examination is unremarkable. The electrocardiogram in the office shows no abnormalities. The patient is referred for an outpatient exercise stress test. During the fifth minute of the Bruce protocol, the electrocardiogram illustrates a 3 mm ST segment depression in leads V3 - V5. What is the most appropriate next step in the management of this patient? B. C. D. E. A. Cardiac catheterization Stress imaging (stress perfusion scan) Repeat stress test with right-sided leads Observation with repeat stress test in 3 months Start on aspirin, beta-blocker, and repeat stress in 3 months

This subscription is licensed to user: roopika only User ID: 123489 Explanation: The American College of Cardiology/American Heart Association (ACC/AHA) defines a positive exercise stress test as 1 mm or more of downsloping or flat ST segment depression during exercise or during the recovery period. The patient in the above vignette has the classic symptoms of stable exertional angina with a positive stress test (3 mm ST segment depression during exercise). The presence of ST depression during exercise denotes the presence of critical coronary lesions which necessitates cardiac catheterization. In this case, therefore, the appropriate next step is to recommend cardiac catheterization to define the anatomic coronary artery lesions and possible intervention with balloon angioplasty and stenting.

(Choice B) The principles and practice of stress testing are based on the relative development of manifestations of myocardial ischemia as the stress to the myocardium is increased. Regional wall motion defects are the first to appear (detected by stress echocardiography). This is followed by significant perfusion defects (detected by perfusion scans), pulmonary capillary wedge elevation, ST segment changes on electrocardiogram, and finally, anginal pain. Stress imaging should therefore be pursued if there are no EKG changes during or after the stress test. It is also the modality of choice in patients with bundle branch block, WPW syndrome, paced ventricular rhythm, and baseline ST changes before exercise (ventricular hypertrophy, digitalis therapy, etc.). In this case, the presence of ST segment changes points to significant coronary artery disease, and there is no further need for stress imaging. (Choice C) According to some studies, right-sided leads improve the sensitivity of exercise stress testing; however, this practice is not routinely indicated in all patients. (Choice D)Observation and repeat stress testing is inappropriate in this patient, who has significant clinical symptoms and a positive stress test. (Choice E) There is no doubt that this patient will need medical therapy; however, cardiac catheterization remains as the most appropriate next step in management. Educational Objective: A positive stress test result in a patient with symptoms warrants further intervention, preferably with a coronary angiogram. Nuclear scans should be performed in subsets of patients with baseline ST segment changes. 65% of people answered this question correctly; This subscription is licensed to user ID: 123489 only

A 55-year-old Caucasian woman comes to the office for a routine follow-up examination. Her only complaints at this time regard some aches and pains in various muscles. She has been diagnosed with irritable bowel syndrome, fibromyalgia, depression, and rheumatic heart disease. Current medications include fluoxetine and ibuprofen. Vital signs include temperature of 37.0C (98.6F), blood pressure of 128/86 mm Hg, pulse of 80/min, and respirations of 14/min. Heart auscultation reveals a low-pitched, rumbling diastolic murmur that is preceded by an opening snap. The affected cardiac valve is best auscultated at which of the following regions? A. Between the second and third intercostal spaces at the right sternal border B. Between the second and third intercostal spaces at the left sternal border C. Between the fifth and sixth intercostal spaces at the left sternal border D. Between the fifth and sixth intercostal spaces at the right sternal border

E. line

Between the fifth and sixth intercostal spaces in the left mid-clavicular

This subscription is licensed to user: roopika only User ID: 123489 Explanation: Mitral stenosis (MS) arises from a thickening of the mitral valve leaflets that obstructs blood flow from the left atrium to the left ventricle. This obstruction results in increased pressure within the left atrium, the pulmonary vasculature, and the right ventricle and atrium. The most common cause of MS is rheumatic fever, as in this patient; involvement of the mitral valve is thought to occur in up to 90% of individuals with rheumatic heart disease. Mitral stenosis is best heard with the bell of the stethoscope at the cardiac apex, which is located between the fifth and sixth intercostal spaces in the left mid-clavicular line (Choice E). It may also prove helpful if the patient lies in the left lateral decubitus position. The aortic valve is best auscultated between the second and third intercostal spaces at the right sternal border (Choice A). The pulmonic valve is best auscultated between the second and third intercostal spaces at the left sternal border (Choice B). The tricuspid valve is best auscultated between the fifth and sixth intercostal spaces at the left sternal border (Choice C). No valve is best auscultated between the fifth and sixth intercostal spaces at the right sternal border (Choice D). Educational Objective: Mitral stenosis is best heard with the bell of the stethoscope at the cardiac apex, which is located between the fifth and sixth intercostal spaces in the left mid-clavicular line. It may also prove helpful if the patient lies in the left lateral decubitus position. 66% of people answered this question correctly; This subscription is licensed to user ID: 123489 only

A 32-year-old African-American woman comes to the office with a complaint of progressive exertional dyspnea. She has a history of seizure disorder, hypertension, idiopathic dilated cardiomyopathy, and angioedema with angiotensin converting enzyme inhibitor use. Her ejection fraction on a recent 2 D-echocardiogram was estimated to be 15 - 20%. Her medications include phenytoin (Dilantin), frusemide, carvedilol, spironolactone, and digoxin. Her blood pressure is 140/90 mmHg, pulse is 65/min, and

respirations are 18/min. Her pulse oximetry is 98% at room air. Physical examination reveals normal jugular venous pressure, clear lung fields, normal first and second heart sounds, and a distinct third heart sound. She is started on a hydralazine and isosorbide dinitrate combination for congestive heart failure. Which of the following statements is true regarding the initiation of this combination in this patient? A. months B. The patient should report any new onset of flu-like symptoms C. It should be stopped for 3 months prior to planning a pregnancy D. The patient should get her liver function test done at baseline and then every 6 months while on the drug E. Her dilantin dose should be increased as hydralazine can lower seizure threshold This subscription is licensed to user: roopika only User ID: 123489 Explanation: The hydralazine and isosorbide combination is frequently used in the management of congestive heart failure in patients intolerant of ACE inhibitors. Patients should be informed regarding the possible adverse reactions that may result from the use of these drugs, and instructed to stop the medication if they develop any of the these reactions. The development of drug-induced lupus-like syndrome has been associated with the use of hydralazine. Other drugs that have been associated with lupus include procainamide, penicillamine, isoniazid, minocycline, diltiazem, methyldopa, chlorpromazine, and interferon-alfa. Patients with drug-induced lupus can develop a variety of flu-like symptoms such as fever, malaise, arthralgias, myalgias, and maculopapular facial rash. They can also develop lymphadenopathy, splenomegaly, pleurisy, and pericarditis. An important immunological marker of drug-induced lupus is the presence of antihistone antibodies. (Choice A) Hydralazine use does not lead to weight loss. On the contrary, its use can cause weight gain by causing fluid and sodium retention. The dose of diuretics may have to be increased to offset the fluid retention caused by hydralazine use. (Choice C) Hydralazine is one of the few antihypertensive medications that can be used safely in a pregnant patient. Some of the other drugs that are safe in pregnancy are methyldopa, labetalol, and long-acting calcium channel blockers. (Choice D) Hydralazine has no hepatic toxicity; therefore, monitoring of liver function tests is not required. (Choice E) Hydralazine does not lower the seizure threshold, and can be safely used in patients with a history of seizure disorder. It may lead to significant weight loss in the first few

Educational Objective: It is important to recognize the syndrome of drug-induced lupus caused by hydralazine. Stopping the medication is usually sufficient to resolve the symptoms. 28% of people answered this question correctly; This subscription is licensed to user ID: 123489 only

Das könnte Ihnen auch gefallen